MCAT Organic Chemistry Flashcards

You may prefer our related Brainscape-certified flashcards:
1
Q

Why is the alpha-anomer of d-glucose less likely to form than the beta-anomer?

A) The beta-anomer is preferred for metabolism
B) The beta-anomer undergoes less electron repulsion.
C) The alpha-anomer is the more stable anomer.
D) The alpha-anomer forms more in L-glucose.

A

B

The hydroxyl group on the anomeric carbon of the beta-anomer is equatorial, thereby creating less steric hinderance than the alpha-anomer, which has the hydroxyl group of anomeric carbon in the axial position.

How well did you know this?
1
Not at all
2
3
4
5
Perfectly
2
Q

Ethanol has been popular with humans for more than 10,000 years. Moreover, many animals are known to seek out rotten fruits that have fermented even moderate levels of alcohol. Which of the following agents would oxidize a primary alcohol to an aldehyde?

A) Pyridinium chlorochromate
B) Na2Cr2O7
C) K2CrO7
D) CrO3, H2SO4
E) None of the above
A

A)

Pyridinium Chlorochromate (PCC) converts a primary alcohol to an aldehyde because it lacks the water necessary to hydrate the otherwise easily hydrated aldehyde. Both Na2Cr2O7 and K2Cr2O7 will produce a carboxylic acid. CrO3, H2SO4 will also produce a carboxylic acid.

How well did you know this?
1
Not at all
2
3
4
5
Perfectly
3
Q

The benzene ring typically undergoes:

A) Nucleophilic addition
B) Nucleophilic substitution
C) Electrophilic addition
D) Electrophilic substitution

A

D

Substitution reactions are preferred because they preserve the very stable aromatic structure. Electrophiles can join the ring because of the resonance stabilization of the intermediate ion.

How well did you know this?
1
Not at all
2
3
4
5
Perfectly
4
Q

Reaction of alcohols with nucleophiles often requires the presence of acid because:

A) Alcohols only react with hydrogen halides
B) -OH is a poor leaving group and therefore must be pronated to form a better leaving group, -OH2+
C) Acids prevent rearrangement of the carbocation
D) Acids prevent bond cleavage

A

B

The hydroxide group is a strong base and therefore a poor leaving group. It must be pronated to form a weak base, H2O, so that it can leave the molecule.

How well did you know this?
1
Not at all
2
3
4
5
Perfectly
5
Q

What term accurately describes the structural relationship between (1R,2S)-1,2-dibromocyclopentane and (1R,2R)-1,2-dibromocyclopentane?

A) Conformers
B) Enantiomers
C) Diastereomers
D) Constitutional isomers

A

C

When drawing out the structures, one can see that the basic skeletal arrangements are identical, i.e. stereoisomers (not constitutional isomers). We can dismiss conformers, since the structures cannot be interconverted exclusively by rotations about formally single bonds. The compounds are diastereomers, a class of stereoisomers that are non-superimposable, nonmirror images of one another. More specifically, the compounds are epimers: in chemistry, epimers are diastereomers that differ in configuration of only one stereogenic center.

(Difference between molecules: both have pentane ring, one has Br towards and away, the other has both Br atoms toward).

How well did you know this?
1
Not at all
2
3
4
5
Perfectly
6
Q

What is the major product of the reaction below, and what is the mechanism by which it is produced?
(CH3CH2)3CBr + F– –> ; temperature: 500C; solvent: water

A) (CH3CH2)3CF; SN1
B) (CH3CH2)3CF SN2
C) CH3CH=C(CH2CH3)2; E2
D) CH3CH=C(CH2CH3)2; E1

A

A

This is an example of a SN1 reaction where first step is Br (good leaving group) dissociates to produce a carbocation (stabilized by the polar protic solvent, water). The tertiary carbocation intermediate is very stable, especially in the highly polar protic solvent water. The carbocation then combines with the strong nucleophile, the F- ion. The original substituent (Br) must be a better leaving group than the nucleophile (F-) in order for the reaction to occur). C and D are wrong because this is not an elimination reaction. B is wrong because this is not an SN2 reaction, due to the highly hindered substrate, and the type of solvent used in the reaction (SN2 reactions favor polar aprotic solvents, not polar protic solvents).

How well did you know this?
1
Not at all
2
3
4
5
Perfectly
7
Q

Heterocyclic amines are found in the base of DNA. Their most important property includes their basicity due to the presence of nitrogen. Which of the following statements is true about the extra pair of electrons in pyrrole and pyridine? (Google search these amines to see structures.)

A) In pyrrole, an sp2 orbital contains the pair of electrons, while in pyridine, they are involved in the pi cloud
B) In both pyrrole and pyridine, an sp2 orbital contains the pair of electrons
C) In both pyrrole and pyridine, they are invovled in the pi cloud
D) In pyrrole, they are invovled in the pi cloud, while in pyridine, an sp2 orbital contains the pair of electrons

A

D

In pyrrole, the sigma bonds around nitrogen are formed by sp2 hybrid orbitals, not sp3. A nitrogen electron participates in each of the sigma bonds. The remaining nitrogen electrons are in a nitrogen 2p orbital which overlaps with the carbon 2p orbitals in the ring to form pi bonds and the cyclic compound, delocalizing the lone pair. In pyridine, the nitrogen atom donates one electron to each of the two sigma bonds by use of sp2 orbitals. The third sp2 orbital contains two electrons, which are for acids. One electron of nitrogen is present in a p orbital, which interacts with the carbon p orbitals to form the pi bond.

How well did you know this?
1
Not at all
2
3
4
5
Perfectly
8
Q

Heterocyclic amines are a very important type of amino group. One use of them includes being the base of DNA. The most important property however is their basicity due to nitrogen.
Pyrrole undergoes electrophilic substitution at position 2 because:
(Google search these amines to see structures.)

A) The carbocation intermediate has more resonance structures than the intermediate formed when the electrophile attacks position 3
B) The inductive effect of nitrogen is greater
C) The aromatic ring is maintained in the carbocation
D) Resonance structures cannot be drawn for the carbocation intermediate formed by the attack at position 3

A

A

Attack at position 2 gives three resonance structures greatly adding to stability of the molecule (positive charge on nitrogen, then position 3, and then position 5). Position 3 only has two resonance structures.

How well did you know this?
1
Not at all
2
3
4
5
Perfectly
9
Q

Which of the following statements about 3-bromo-1,1-dimethylcyclobutane are true?

I. It is saturated
II. It is a planar molecule.
III. It has no ring strain

A) I only
B) II only
C) III only
D) I and III

A

A

A saturated molecule is one where all the valences of the atoms are satisfied by single covalent bonds. In other words, there are no double bonds. Statement I is true.

A planar molecule is a flat one. In this case, we have two opposite corners in a tetrahedral shape. II is not true.

Ring strain is minimized in a cyclohexane and maximized in a cyclopropane, but it is always present in a cyclic molecule. III is not true. Thus the answer is A.

How well did you know this?
1
Not at all
2
3
4
5
Perfectly
10
Q

What is the major product of the reaction below, and what is the mechanism by which it is produced?
(CH3CH2)3CBr + F– –> ; temperature: 500C; solvent: water

A) (CH3CH2)3CF; SN1
B) (CH3CH2)3CF SN2
C) CH3CH=C(CH2CH3)2; E2
D) CH3CH=C(CH2CH3)2; E1

A

A

This is an example of a SN1 reaction where first step is Br (good leaving group) dissociates to produces a carbocation (stabilized by the polar protic solvent, water). The tertiary carbocation intermediate is very stable, especially in the highly polar protic solvent water. The carbocation then combines with the strong nucleophile, the F- ion. (the original substituent (Br) must be a better leaving group than the nucleophile (F-) in order for the reaction to occur). C and D are wrong because this is not an elimination reaction. B is wrong because this is not an SN2 reaction, due to the highly hindered substrate, and the type of solvent used in the reaction (SN2 reactions favor polar aprotic solvents, not polar protic solvents).

How well did you know this?
1
Not at all
2
3
4
5
Perfectly
11
Q

The hybridization state and shape of a carbocation is:

A) sp and linear
B) sp2 and trigonal planar
C) sp3 and tetrahedral
D) sp3 and trigonal planar

A

B

On the surface: A carbocation is bonded to 3 substituents and so it is in the middle of a flat triangle: trigonal planar.

When carbon has:
4 subtituents: sp3 and tetrahedral (109.5º)
3 subtituents: sp2 and trigonal planar (120º)
2 subtituents: sp and linear (180º)

Going Deeper: There are three bonding electron pairs around the carbon atom, and these can be placed as far apart as possible in an sp2 trigonal planar framework. The empty orbital, which results in the charge, is the 2pz atomic (unhybridized) orbital.

How well did you know this?
1
Not at all
2
3
4
5
Perfectly
12
Q

(Fill in the blanks) Benzoic acid can be extracted into water from an organic solvent using _________ and then removed from the water into an organic solvent by the addition of _________.

A) Sodium bicarbonate / sodium hydroxide
B) Hydrochloric acid / sodium hydroxide
C) Hydrochloric acid / sulfuric acid
D) Sodium hydroxide / hydrochloric acid

A

D

Benzoic acid, an organic compound, is insoluble in water. Extraction into water can be achieved with aqueous sodium hydroxide (deprotonation -> polar sodium benzoate, now soluble in water). Transfer of the benzoate back into organic solution is accomplished by treatment with hydrochloric acid (protonation -> less polar benzoic acid).

How well did you know this?
1
Not at all
2
3
4
5
Perfectly
13
Q

A high intensity peak in the mass spectrum of 2,2-dimethylpentane is observed at m/z 57. What chemical species does this peak represent?

A) Methyl cation
B) tert-butyl carbocation
C) n-propyl carbocation
D) Ethyl carbocation

A

B

The most stable fragment likely formed from 2,2-dimethylpentane fragmentation is t-butyl species.

T-butyl species = 57 m/z on mass spectrum

How well did you know this?
1
Not at all
2
3
4
5
Perfectly
14
Q

The addition of an acid and heat to 2-oxocyclopentanecarboxylic acid yields a:

A) cyclopentene
B) cyclopentanol
C) cyclopentanone
D) cyclopentanecarboxylic acid

A

C

Beta-keto carboxylic acids undergo decarboxylation through ring formations. The carbonyl bond of the ketone will react with the alcohol group of the carboxylic acid to form a ring and release the carboxylic acid as CO2. The remaining enol will tautomerize into a carbonyl group again, leaving cyclopentanone.

How well did you know this?
1
Not at all
2
3
4
5
Perfectly
15
Q

The data in Table 1 were collected for Reaction I:
2X + Y –> Z

Reaction I
Exp [X] in M [Y] in M Initial Rate of Rxt
1 0.050 0.100 8.5 x 10^-6
2 0.050 0.200 3.4 x 10^-5
3 0.200 0.100 3.4 x 10^-5

What is the rate law for the reaction?

A) Rate = k[X]^2[Y]
B) Rate = k[X]^2[Y]^2
C) Rate = k[X][Y]^2
D) Rate = k[X][Y]

A

C

This is a common MCAT-type question. By looking at Table 1, we can see that when the concentration of X is quadrupled (factor of 41) while [Y] is unchanged (Exp. 1 and 3), the rate is increased by a factor of 4 = 41. Thus the order of the reaction with respect to X is 1 (= “first order with respect to X”).

When the concentration of Y is doubled (factor of 21) while [X] remains the same (Exp. 1 and 2), the rate of reaction is quadrupled (factor of 4 = 22). Thus, the order of reaction with respect to Y is 2 (= “second order with respect to Y”).

The rate equation is Rate = [X][Y]2. Since the overall rate of reaction is the sum of exponents, we can say that the reaction is (1 + 2) a third order reaction overall.

{Notice that the stoichiometric coefficients are not relevant; the order of reaction being based on data.}

Additional learning point: In Organic Chemistry, an SN2 reaction is typical of a second order reaction where the rate must be equally proportional to the concentration of both reactants; for example, first order with respect to the alkyl halide and first order with respect to the nucleophile, so overall second order.

How well did you know this?
1
Not at all
2
3
4
5
Perfectly
16
Q

The anomers of D-(+)-glucose differ in their configuration around which of the following?

A) C-1
B) C-5
C) C-6
D) C-2

A

A

Glucose anomers have a different orientation of the hydroxyl group on the C-1 carbon.

How well did you know this?
1
Not at all
2
3
4
5
Perfectly
17
Q

A radioactive substance A was placed in a lead-lined container and the radiation it emitted was allowed to pass through a small aperture in the container and below a positively charged plate. The path of the particle changed such that it angled up toward the plate. Which of the following could it be?

A) Alpha particle
B) Beta particle
C) Proton
D) Gamma particle

A

B

Since the radioactive emission angled up toward the positive plate, it must be oppositely charged, that is, negatively charged. The only negatively charged species in the answer choices is the beta particle which is, by definition, an electron or a positron. Note that electromagnetic radiation like gamma rays (particles) and infrared radiation are not deflected in electromagnetic fields.

How well did you know this?
1
Not at all
2
3
4
5
Perfectly
18
Q

Heterocyclic amines are a very important type of amino group. One use of them includes being the base of DNA. The most important property however is their basicity due to nitrogen.

Pyridine is a much stronger base than pyrrole because:

A) Pyrrole will lose its aromatic character if nitrogen accepts an acid
B) Pyridine has a pair of electrons in an sp2 orbital that is available for sharing
C) The extra pair of electrons on nitrogen in pyrrole is involved in the pi cloud and therefore less available for sharing
D) All of the above

A

D

In pyrrole, the sigma bonds around nitrogen are formed by sp2 hybrid orbitals, not sp3. A nitrogen electron participates in each of the sigma bonds. The remaining nitrogen electrons are in a nitrogen 2p orbital which overlaps with the carbon 2p orbitals in the ring to form pi bonds and the cyclic compound, delocalizing the lone pair. In pyridine, the nitrogen atom donates one electron to each of the two sigma bonds by use of sp2 orbitals. The third sp2 orbital contains two electrons, which are for acids. One electron of nitrogen is present in a p orbital, which interacts with the carbon p orbitals to form the pi bond.

How well did you know this?
1
Not at all
2
3
4
5
Perfectly
19
Q

Which molecules contain an IR stretching frequency of 1680-1740 cm-1?

I. Alcohols
II. Carboxylic acids
III. Nitriles
IV. Amides

A) I and II
B) II and III
C) II and IV
D) III and IV

A

C

All carbonyl compounds absorb in the region 1760-1665 cm-1 due to the stretching vibration of the C=O bond. Different carbonyl compounds absorb in narrow ranges within this general region. Carboxylic acids and amides contain carbonyl functionality. Alcohols have broad absorption at approx. 3200-3500 cm-1; nitriles show absorptions at approx. 2250 cm-1.

How well did you know this?
1
Not at all
2
3
4
5
Perfectly
20
Q

In the phrase “SN1 Reaction”, what does the “1” indicate?

A) The equilibrium constant
B) The number of reactants involved
C) The stereochemical outcome
D) The rate order

A

D

“SN” stands for nucleophilic substitution and the “1” represents the fact that the rate-determining step is unimolecular.

How well did you know this?
1
Not at all
2
3
4
5
Perfectly
21
Q

In proton NMR spectroscopy, which compounds or functional groups show a characteristic peak in the range 9-10 ppm?

A) Halogenated alkanes
B) Aldehydes
C) Alcohols
D) Aromatic compounds

A

B

Nuclear magnetic resonance (NMR) is concerned with the magnetic properties of certain nuclei. Anything that removes electron density from around a nucleus deshields it and shifts the absorption to higher frequency. The more electronegative the atoms that are attached to the nucleus, the higher the absorption frequency. Thus, NMR is able to provide information on the types of functional groups present as each has a characteristic chemical shift range. ALDEHYDES HAVE A VERY CHARACTERSTIC SHIFT OF 9-10 PPM.

How well did you know this?
1
Not at all
2
3
4
5
Perfectly
22
Q

Choose the molecules containing an IR stretching frequency of 3500-3000 cm-1.

I. Amines
II. Carboxylic Acids
III. Phenols
IV. Alcohols

A) (I) and (II)
B) (II) and (III)
C) (III) and (IV)
D) All of them

A

D

The range 3500-3000 cm-1 is typically associated with stretches between heteroatoms and a hydrogen atom, for example NH and OH bonds. They may appear broad, intense and jagged (COOH), broad and smooth (OH) or less intense and sharper (NH). The preceding sentence is likely more than you would need to know for the MCAT. In general, for MCAT purposes, it is useful to remember APPROX. 3300 FOR HYDROXYL AND APPROX. 1700 FOR CARBONYL (C=O).

How well did you know this?
1
Not at all
2
3
4
5
Perfectly
23
Q

Infrared spectroscopy is often used to determine the presence of a carbonyl group in a compound. It is a strong band appearing at about 1700 cm^-1 in aldehydes, ketones, and carboxylic acids and their derivatives. A carboxylic acid will show another carbon oxygen stretching band at:

A) A higher frequency
B) A lower frequency
C) The same frequency
D) A frequency in the ultraviolet region of the electromagnetic spectrum

A

B

A C-O single bond is weaker than a double bond. Therefore, less energy is needed to induce the stretching vibration. Remember reciprocal centimeters is a measure of frequency and energy is directly proportional to the frequency.

How well did you know this?
1
Not at all
2
3
4
5
Perfectly
24
Q

Which reagent would you choose to convert hexan-1-ol to hexanal?

A) PCC/CH2Cl2
B) DIBAL/diethyl ether
C) KMnO4/aqueous H2SO4/acetone
D) K2Cr2O7/aqueous H2SO4/acetone

A

A

Pyridinium chlorochromate (PCC) is a mild oxidant, which converts primary and secondary alcohols to aldehydes and ketones respectively. KMnO4 and K2Cr2O7 are harsher oxidants, and would further oxidize primary alcohols to the corresponding carboxylic acids. Di-isobutylaluminum hydride (DIBAL) is a reducing agents.

How well did you know this?
1
Not at all
2
3
4
5
Perfectly
25
Q

Which statement below about recrystallization is NOT correct?

A) The method relies on the fact that solids are more soluble in a cold solvent than they are in the same amount of hot solvent
B) Precipitation is initiated by adding a seed crystal of the compound being purified
C) Scratching the side of the vessel at the air/solution interface can induce crystal nucleation
D) Slowly cooling the solution promotes precipitation of the compound

A

A

This statement is reversed: The process of recrystallization involves dissolution of the solid in appropriate solvent at a high temperature and the subsequent re-formation of the crystals upon cooling, so that any impurities remain in solution.

How well did you know this?
1
Not at all
2
3
4
5
Perfectly
26
Q

_____________ refers to the method by which an optically reactive reagent or catalyst is used to transform an optically inactive starting material into an optically active product.

A) Racemization
B) Enantioenrichment
C) Chiral chromatography
D) Asymmetric induction

A

D

Racemization refers to the conversion of an optically active substance to a racemic form (has equal amounts of left- and right-handed enantiomers of a chiral molecule). Enantioenrichment refers to the act of enriching an already existing mixture of stereoisomers such that it contains more than 50% of one enantiomer. Chiral chromatography involves the separation of stereoisomers using a chiral stationary phase, which does not involve chemical transformations. “Optical reduction” as a technique or phrase does not exist.

How well did you know this?
1
Not at all
2
3
4
5
Perfectly
27
Q

Which of the following solutions/compounds is/are optically inactive?

A) A racemic mixture showing no rotation of plane polarized light
B) A meso compound
C) A 50-50 mixture of a D and L form of a substance
D) All of the above

A

D

All the mentioned forms of compounds in this question show no rotation of plane polarized light. Racemic mixtures showing no rotation of plane polarized light, like a 50-50 mix of D and L compounds, are exact racemic mixtures of two enantiomeric forms of a compound thus cancelling the polarization effect of plane polarized light allowing for optical isomerization. A meso compound is superposable on its mirror image meaning that the molecule is achiral (i.e. optically inactive) though it may contain chiral centers.

How well did you know this?
1
Not at all
2
3
4
5
Perfectly
28
Q

The technique of fractional distillation is most effective for:

A) Separating solids on the basis of their different solubilities in a particular liquid
B) Identifying the functional groups in mixtures of more than one liquid
C) Separating liquids based on their different boiling points
D) Determining the boiling points of a variety of liquids

A

C

Using fractional distillation one can increase the temperature of a liquid mixture and as the different liquids vaporize at their respective boiling point temperatures one can collect the vapor by condensing it and collecting the desired liquid at the corresponding temperature.

How well did you know this?
1
Not at all
2
3
4
5
Perfectly
29
Q

Which angle is common to tetrahedral molecules?

A) 109.5°
B) 120°
C) 60°
D) 90°

A

A

Tetrahedral geometry has three-dimensional placement with atoms positioned at 109.5° to each other. Trigonal planar molecules have angles of 120°, interior angles of 60° are associated with strained cyclopropanes bonding, 90° with cyclobutanes and 180° for alkynes.

How well did you know this?
1
Not at all
2
3
4
5
Perfectly
30
Q

Which of the following is the most likely to react with Magnesium (Mg)?

A) O
B) Ca
C) S
D) Cl

A

D

Electronegativity increases to the right of the periodic table, and also from bottom to top. We note that chlorine is the closest to the upper right of the periodic table, meaning it is the most electronegative of the available choices. Thus, it is the most likely to react with Mg, which normally carries a 2+ charge.

How well did you know this?
1
Not at all
2
3
4
5
Perfectly
31
Q

How many orbitals can occupy the 5f subshell?

A) 5
B) 7
C) 10
D) 14

A

B

There is one s orbital, and there are three p orbitals, five d orbitals, and seven f orbitals. Thus, the answer is B. Do not confuse the number of orbitals in a subshell with the number of electrons the subshell can hold. Each orbital can hold two electrons, so the capacity of an nf subshell is 14 electrons.

How well did you know this?
1
Not at all
2
3
4
5
Perfectly
32
Q

Toluene will show peaks in its:

A) IR spectrum only
B) UV and IR spectrum only
C) IR, UV, and NMR spectrum
D) UV spectrum only

A

C

Aromatic systems show strong absorptions in the UV region of the system. C-C and C-H bonds will absorb in the IR region. The NMR will show the inequivalent hydrogens.

How well did you know this?
1
Not at all
2
3
4
5
Perfectly
33
Q

Acetic acid reacts more rapidly with methanol than with ethanol to form an ester. Most likely this is due to:

A) An inductive electronic effect
B) A resonance effect
C) Steric hindrance
D) An ortho effect

A

C

Bulky groups on either the acid or alcohol slow the reaction due to steric hindrance.

How well did you know this?
1
Not at all
2
3
4
5
Perfectly
34
Q

Which of the following lists the correct common names for ethanal, methanal, and ethanol, respectively?

A) Acetaldehyde, formaldehyde, ethyl alcohol
B) Ethyl alcohol, propionaldehyde, isopropyl alcohol
C) Ethyl alcohol, formaldehyde, acetaldehyde
D) Isopropyl alcohol, ethyl alcohol, formaldehyde

A

A

The common name of ethanal is acetaldehyde, the common name of methanal is formaldehyde, and the common name of ethanol is ethyl alcohol. Isopropyl alcohol is the common name of 2-propanol. Propionaldehyde is the common name of propanal.

How well did you know this?
1
Not at all
2
3
4
5
Perfectly
35
Q

Which of the following are considered terminal functional groups?

I. Aldehydes
II. Ketones
III. Carboxylic acids

A) I only
B) III only
C) I and III only
D) I, II, and III

A

C

Aldehydes and carboxylic acids are characterized by their positions at the ends of carbon backbones and are thus considered terminal groups. As a result, the carbons to which they are attached are usually designated carbon 1. Ketones are internal by definition because there must be a carbon on either side of the carbonyl.

How well did you know this?
1
Not at all
2
3
4
5
Perfectly
36
Q

NADH is a coenzyme that releases high-energy electrons into the electron transport chain. It is known as nicotinamide adenine dinucleotide or diphosphopyridine nucleotide. What functional groups exist in this molcule?

I. Phosphate
II. Amide
III. Anhydride

A) I only
B) II only
C) I and II only
D) I, II, and III

A

C

The suffix -amide in nicotinamide indicates that this compound contains an amide functional group. The prefix diphospho- indicates that there are two phosphate groups, as well. Even if we did not know the prefix phospho- from this chapter, we should recognize that nucleotides, mentioned in the name, contain a sugar, a phosphate group, and a nitrogenous base.

How well did you know this?
1
Not at all
2
3
4
5
Perfectly
37
Q

Which of the following are common names for carboxylic acid derivatives?

I. Acetic anhydride
II. Formic acid
III. Methyl formate

A) I and II only
B) I and III only
C) II and III only
D) I, II, and III

A

B

Acetic anhydride is the common name for ethanoic anhydride. Methyl formate is the common name for methyl methanoate; we can infer this from the common root form - and the ester suffix -oate (which is sometimes shortened to -ate for pronunciation purposes). Formic acid is the common name for methanoic acid, but this is a carboxylic acid - not a derivative.

How well did you know this?
1
Not at all
2
3
4
5
Perfectly
38
Q

Consider the name 2,3-diethylpentane. Based on the structure implied by this name, the correct IUPAC name for this molecule is:

A) 2,3-diethylpentane
B) 1,2-diethylbutane
C) 3-ethyl-4-methylhexane
D) 3-methyl-4-ethylhexane

A

C

Draw out the molecule, and you will see that the longest carbon chain with the substituents at the lowest possible carbon numbers is actually different from the one chosen in the original name. The correct IUPAC name for this molecule is 3-ethyl-methylhexane.

How well did you know this?
1
Not at all
2
3
4
5
Perfectly
39
Q

Which of the following does NOT show optical activity?

A) (R)-2-butanol
B) (S)-2-butanol
C) A solution containing 1 M (R)-2-butanol and 2 M (S)-2-butanol
D) A solution containing 2 M (R)-2-butanol and 2 M (S)-2-butanol

A

D

This is a racemic mixture of 2-butanol because it consists of equimolar amounts of (R)-2-butanol and (S)-2-butanol. The (R)-2-butanol molecule rotates the plan of polarized light in one direction, and the (S)-2-butanol rotates it by the same angle but in the opposite direct; as a result, no net rotation of polarized light is observed.

How well did you know this?
1
Not at all
2
3
4
5
Perfectly
40
Q
How many stereoisomers exist for the following aldehyde?
                   O
                    ||
                   C  -  H
                    |
        HO  -  C  -  H
                    |
        HO  -  C  -  H
                    |
        HO  -  C  -  H
                    |
        HO  -  C  -  H
                    |
                   H

A) 2
B) 8
C) 9
D) 16

A

B

The maximum number of stereoisomers of a compound equals 2^n, where n is the number of chiral carbons in the compound. In this molecule, C-1 (the aldehydic carbon) is not chiral, nor is C-5 (because it is attached to two hydrogen atoms). Therefore, with three chiral centers, there are 2^3 = 8 stereoisomers.

How well did you know this?
1
Not at all
2
3
4
5
Perfectly
41
Q

Which of the following compounds is optically inactive?

A)        CH3
             |
     H  -  |  -  Cl
             |
    Cl  -  |  -  H
             |
            CH3
B)        CH3
             |
    Cl  -  |  -  H
             |
     H  -  |  -  Cl
             |
            CH3
C)        CH3
             |
     H  -  |  -  Cl
             |
     H  -  |  -  Cl
             |
            CH3
D)        CH2Cl
             |
     H  -  |  -  Cl
             |
     H  -  |  -  H
             |
            CH3
A

C

This answer choice is an example of a meso compound - a compound that contains chiral centers but has an internal plane of symmetry:

                CH3
                 |
         H  -  |  -  Cl
    -----------|-------------  Plane of
         H  -  |  -  Cl        Symmetry
                 |
                CH3

Owing to this internal plane of symmetry, the molecule is achiral and, hence, optically inactive. Choices A and B are enantiomers of each other and will certainly show optical activity on their own. Choice D is incorrect because it contains a chiral carbon and no internal plane of symmetry, thus, it is optically active as well.

How well did you know this?
1
Not at all
2
3
4
5
Perfectly
42
Q

(+)-Glyceraldehyde and (-)-glyceraldehyde refer to the (R) and (S) forms of 2,3-dihydroxypropanal, respectively. These molecules are considered:

A) Enantiomers
B) Diastereomers
C) Meso compounds
D) Structural isomers

A

A

(+)-Glyceraldehyde and (-)-glyceraldehyde, or (R)- and (S)-2,3-dihydroxypropanal, are enantiomers. Enantiomers are nonsuperimposable mirror images. Each has only one chiral center (C-2), which has the opposite configuration in these two molecules.

How well did you know this?
1
Not at all
2
3
4
5
Perfectly
43
Q

Consider (E)-2-butene and (Z)-2-butene. This is a pair of what type(s) of isomers?

I. Cis-trans isomers
II. Diastereomers
III. Enantiomers

A) I only
B) II only
C) I and II only
D) I and III only

A

C

(E)-2-butene can also be called trans-2-butene; (Z)-2-butene can also be called cis-2-butene. As such, they are cis-trans isomers. Remember that cis-trans isomers are a subtype of diastereomers in which the position of substituents differs about an immovable bond. Diastereomers are molecules that are non-mirror-image stereoisomers (molecules with the same atomic connectivity). These are not enantiomers because they are not mirror images of each other.

How well did you know this?
1
Not at all
2
3
4
5
Perfectly
44
Q

3-methylpentant and hexane are related in that they are:

A) Enantiomers
B) Diastereomers
C) Constitutional isomers
D) Conformational isomers

A

C

Because they have the same molecular formula but different connectivity, 3-methylpentane and hexane are constitutional isomers.

How well did you know this?
1
Not at all
2
3
4
5
Perfectly
45
Q

(R)-2-chloro-(S)-3-bromobutane and (S)-2-chloro-(S)-3-bromobutane are:

A) Enantiomers
B) Diastereomers
C) Meso compounds
D) The same molecule

A

B

These two molecules are stereoisomers of one another, but are NOT nonsuperimposable mirror images. Therefore, they are diastereomers. Note that these molecules differ by at least one, but not all, chiral carbons.

How well did you know this?
1
Not at all
2
3
4
5
Perfectly
46
Q

A scientist takes a 0.5g/mol solution of an unknown pure dextrorotatory organic molecule and places it in a test tube with a diameter of 1cm. He observes that a plane of polarized light is rotated 12 degrees under these conditions. What is the specific rotation of this molecule?

A) -240 degrees
B) -24 degrees
C) +24 degrees
D) +240 degrees

A

D

Remember that the equation for specific rotation is [alpha] = a(obs)/(c x l). In this example a(obs) is +12 degrees (remember that dextrorotatory, or clockwise, rotation is positive), c = 0.5g/ml, l = 1cm = 0.1dm. Remember that path length is ALWAYS measured in decimeters when calculating specific rotation. Therefore, the specific rotation can be calculated as:

[alpha] = +12 / (0.5g/ml x 0.1 dm) = +240 degrees.

How well did you know this?
1
Not at all
2
3
4
5
Perfectly
47
Q

Omeprazole is a proton pump inhibitor commonly used in gastroesophageal reflux disease. When omeprazole, a racemic mixture, went off-patent, pharmaceutical companies began to manufacture esomeprazole, the (S)-enantiomer of omeprazole, by itself. Given 1 M solution of omeprazole and esomeprazole, which solution(s) would likely exhibit optical activity?

A) Omeprazole only
B) Esomeprazole
C) Both omeprazole and esomeprazole
D) Neither omeprazole nor esomeprazole

A

B

Racemic mixtures like omeprazole contain equimolar amounts of two enantiomers and thus have no observed optical activity. Each of the two enantiomers causes rotation in opposite directions, so their effects cancel out. Esomeprazole only contains one of the two enantiomers and thus should cause rotation of plane-polarized light.

How well did you know this?
1
Not at all
2
3
4
5
Perfectly
48
Q

(2R,3S)-2,3-dihydroxybutanedioic acid and (2S,3R)-2,3-dihydroxybutanedioic acid are:

I. Meso compounds
II. The same molecule
III. Enantiomers

A) I only
B) III only
C) I and II only
D) I and III only

A

C

Draw out these structures. The two names describe the same molecule, which also happens to be a meso compound because it contains a plane of symmetry. These compounds are not enantiomers because they are superimposable mirror images of one another, NOT nonsuperimposable mirror images. These compounds are better termed meso-2,3-dihydroxybutanedioic acid:

                              COOH
                               |
                       H  -  |  -  OH
                 ------------|--------------
                       H  -  |  -  OH
                               |
                              COOH
How well did you know this?
1
Not at all
2
3
4
5
Perfectly
49
Q

If the methyl groups of butane are 120 degrees apart, as seen in a Newman projection, this molecule is in its:

A) Highest-energy gauche form
B) Lowest-energy staggered form
C) Middle-energy eclipsed form
D) Highest-energy eclipsed form

A

C

In butane, the position at which the two methyl groups are 120 degrees apart is an eclipsed conformation. This has a moderate amount of energy, although not as high as a totally eclipsed conformation in which the two methyl groups are 0 degrees apart.

How well did you know this?
1
Not at all
2
3
4
5
Perfectly
50
Q

Within one principle energy level, which subshell has the least energy?

A) s
B) p
C) d
D) f

A

A

The energies of the subshells within a principle quantum number are as follows: s < p < d < f.

How well did you know this?
1
Not at all
2
3
4
5
Perfectly
51
Q

Which of the following compounds possesses at least one sigma bond?

A) CH4
B) C2H2
C) C2H4
D) All of the above contain at least one sigma bond

A

D

All single bonds are sigma bonds; double and triple bonds each contain one sigma bond and one or two pi bonds, respectively. The compounds CH4, C2H2, and C2H4 all contain at least one single bond and therefore contain at least one sigma bond.

How well did you know this?
1
Not at all
2
3
4
5
Perfectly
52
Q

A carbon atom participates in one double bond. As such, this carbon contains orbitals with:

A) Hybridization between the s-orbital and the one p-orbital
B) Hybridization between the s-orbital and two p-orbitals
C) Hybridization between the s-orbital and three p-orbitals
D) Unhybridized s character

A

B

In a carbon with one double bond, sp2 hybridization occurs - that is, one s-orbital hybridizes with two p-orbitals to form three sp2-hybridized orbitals. The third p-orbital of the carbon atom remains unhybridized and takes part in the formation of the pi bond of the double bond. Although there is an unhybridized p-orbital, there are no unhybridized s-orbitals.

How well did you know this?
1
Not at all
2
3
4
5
Perfectly
53
Q

The hybridization of the carbon and nitrogen atoms in CN- are:

A) sp3 and sp3, respectively
B) sp3 and sp, respectively
C) sp and sp3, respectively
D) sp and sp, respectively

A

D

The carbon and nitrogen atoms are connected by a triple bond in CN- (:C≡N:-). A triple-bonded atom is sp hybridized; one s-orbital hybridizes with one p-orbital to form two sp-hybridized orbitals. The two remaining unhybridized p-orbitals take part in the formation of two pi bonds.

How well did you know this?
1
Not at all
2
3
4
5
Perfectly
54
Q

Which of the following hybridizations does the Be atom in BeH2 assume?

A) sp
B) sp2
C) sp3
D) sp3d

A

A

Beryllium has only two electrons in its valence shell. When it bonds to two hydrogens, it requires two hybridized orbitals, meaning that its hybridization must be sp. Note that the presence of only single bonds does not mean that the hybridization must be sp3; this is a useful assumption for carbon, but does not apply to beryllium because of its smaller number of valence electrons. The two unhybridized p-orbitals around beryllium are empty in BeH2, which takes on the linear geometry characteristic of sp-hybridized orbitals.

How well did you know this?
1
Not at all
2
3
4
5
Perfectly
55
Q

Two atomic orbitals may combine to form:

I. A bonding molecular orbital
II. An antibonding molecular orbital
III. Hybridized orbitals

A) I only
B) III only
C) I and II only
D) I, II and III

A

D

When atomic orbitals combine, they form molecular orbitals. When two atomic orbitals with the same sign are added head-to-head or tail-to-tail, they form bonding molecular orbitals. When two atomic orbitals with opposite signs are added head-to-head or tail-to-tail, they form antibonding molecular orbitals. Atomic orbitals can also hybridize, forming sp3, sp2 or sp orbitals.

How well did you know this?
1
Not at all
2
3
4
5
Perfectly
56
Q

Molecular orbitals can contain a maximum of:

A) One electron
B) Two electrons
C) Four electrons
D) 2n^2 electrons, where n is the principle quantum number of the combining atomic orbitals

A

B

Like atomic orbitals, molecular orbitals each can contain a maximum of two electrons with opposite spins. The 2n^2 rule in Choice D refers to the total number of electrons that can exist in a given energy shell, not in a molecular orbital.

How well did you know this?
1
Not at all
2
3
4
5
Perfectly
57
Q

Pi bonds are formed by which of the following orbitals?

A) Two s-orbitals
B) Two p-orbitals
C) One s-orbital and one p-orbital
D) Two sp2-hybridized orbitals

A

B

Pi bonds are formed by the parallel overlap of unhybridized p-orbitals. The electron density is concentrated above and below the bonding axis. A sigma bond, on the other hand, can be formed by the head-to-head overlap of two s-orbitals or hybridized orbitals. In a sigma bond, the density of the electrons is concentrated between the two nuclei of the bonding atoms.

How well did you know this?
1
Not at all
2
3
4
5
Perfectly
58
Q

How many sigma bonds and pi bonds are present in the following compound?

                          H         O
                           |        //
                  H  -  C  -  C
                           |        \
                          H        H

A) Six sigma bonds and one pi bond
B) Six sigma bonds and two pi bonds
C) Five sigma bonds and one pi bond
D) Five sigma bonds and two pi bonds

A

A

Each single bond has one sigma bond, and each double bond has one sigma and one pi bond. In this question, there are five single bonds (five sigma bonds) and one double bond (one sigma bond and one pi bond), which gives a total of six sigma bonds and one pi bond.

How well did you know this?
1
Not at all
2
3
4
5
Perfectly
59
Q

The four C-H bonds of CH4 point toward the vertices of a tetrahedron. This indicates that the hybridization of the carbon atom in methane is:

A) sp
B) sp2
C) sp3
D) sp3d

A

C

The four bonds point to the vertices of a tetrahedron, which means that the angle between two bonds is 109.5 degrees, a characteristic of sp3 orbitals. Hence, the carbon atom of CH4 is sp3-hybridized.

How well did you know this?
1
Not at all
2
3
4
5
Perfectly
60
Q

Why is a single bond stronger than a pi bond?

I. Pi bonds have greater orbital overlap
II. s-orbitals have more overlap than p-orbitals
III. sp3 hybridization is always unstable

A) I only
B) II only
C) I and III only
D) II and III only

A

B

Bond strength is determined by the degree of orbital overlap; the greater the overlap, the greater the bond strength. A pi bond is weaker than a single bond because there is significantly less overlap between the unhybridized p-orbitals of a pi bond (due to their parallel orientation) than between the s-orbitals or hybrid orbitals of a sigma bond. sp3-hybridized orbitals can be quite stable, as evidenced by the number of carbon atoms with this hybridization forming stable compounds.

How well did you know this?
1
Not at all
2
3
4
5
Perfectly
61
Q

The p character of the bonds formed by the carbon atom in HCN is:

A) 25%
B) 50%
C) 67%
D) 75%

A

B

The carbon bond in hydrogen cyanide (H-C≡N:) is triple-bonded, and because triple bonds require two unhybridized p-orbitals, the carbon must be sp-hybridized; sp-hybridized orbitals have 50% s character and 50% p character.

How well did you know this?
1
Not at all
2
3
4
5
Perfectly
62
Q

A resonance structure describes:

I. The hybrid of all possible structures that contribute to electron distribution
II. A potential arrangement of electrons in a molecule
III. The single form that the molecule most often takes

A) I only
B) II only
C) I and II only
D) I, II, and III

A

B

A resonance structure describes an arrangement of electrons in a molecule. Different resonance structures can be derived by moving electrons in unhybridized p-orbitals throughout a molecule containing conjugated bonds. In molecules that contain multiple resonance structures, some are usually more stable than others; however, each resonance structure is not necessarily the most common form a molecule takes, eliminating statement III. Statement I has reversed the terminology for resonance structures: the electron density in a molecule is the weighted average of all possible resonance structures, not the other way around.

How well did you know this?
1
Not at all
2
3
4
5
Perfectly
63
Q

An electron is known to be in the n = 4 shell and the l = 2 subshell. How many possible combinations of quantum numbers could this electron have?

A) 1
B) 2
C) 5
D) 10

A

D

An electron in the n = 4 shell and the l = 2 subshell can have five different values for ml; -2, -1, 0, 1, or 2. In each of these orbitals, electrons can have positive or negative spin. Thus, there are 5 x 2 = 10 possible combinations of quantum numbers for this electron.

How well did you know this?
1
Not at all
2
3
4
5
Perfectly
64
Q

Compared to single bonds, triple bonds are:

A) Weaker
B) Longer
C) Made up of fewer sigma bonds
D) More rigid

A

D

Pi bonds do not permit free rotation, unlike sigma bonds; this makes triple bonds more rigid than single bonds. Triple bonds are stronger and shorter bonds than single bonds. Both single and triple bonds contain one sigma bond.

How well did you know this?
1
Not at all
2
3
4
5
Perfectly
65
Q

Which of the following are Lewis bases?

I. Ag+
II. H2O
III. NH3

A) I only
B) I and II only
C) II and III only
D) I, II, and III

A

C

NH3 and H2O are Lewis bases because nitrogen and oxygen can donate lone pairs. Ag+ is a Lewis acid because it can accept a lone pair into an unoccupied orbital.

How well did you know this?
1
Not at all
2
3
4
5
Perfectly
66
Q

Rank the following in order of decreasing nucleophilicity in an aprotic solvent: RO-, RCOOH, ROH, HO-

A) RCOOH > ROH > RO- > HO-
B) HO- > ROH > RO- > RCOOH
C) RO- > HO- > ROH > RCOOH
D) RCOOH > RO- > HO- > ROH

A

C

Remember, good nucleophiles tend to have lone pairs or pi bonds and are negatively charged or polarized. Alkoxide (OR-) and hydroxide (OH-) anions are strong nucleophiles. Alcohol (ROH) and carboxylic acids (RCOOH) are weak nucleophiles. The alkyl group of an alkoxide anion donates additional electron density, making it more reactive than the hydroxide ion. The carboxylic acid contains more electron-withdrawing oxygen atoms than the alcohol, making it less nucleophilic.

How well did you know this?
1
Not at all
2
3
4
5
Perfectly
67
Q

Rank the following in order of decreasing electrophilicity; CR3+, CH3OH, CH3OCH3, CH3Cl

A) CH3OCH3 > CR3+ > CH3OH > CH3Cl
B) CR3+ > CH3OH > CH3OCH3 > CH3Cl
C) CH3OCH3 > CH3Cl > CR3+ > CH3OH
D) CR3+ > CH3Cl > CH3OH > CH3OCH3

A

D

Good electrophiles are positively charged or polarized. CH3+ is a tertiary carbocation; it has a positive charge, which makes it very electrophilic. CH3Cl and CH3OH are both polarized; however, the leaving groups differ between these two. Cl- is a weaker base than OH- (HCl is a stronger acid than H2O). As such, Cl- will be more stable in solution than OH-, which increases the electrophilic reactivity of CH3Cl above CH3OH. CH3OCH3 has a much less stable leaving group, CH3O-, and is therefore significantly less electrophilic.

How well did you know this?
1
Not at all
2
3
4
5
Perfectly
68
Q

Rank the following in order of decreasing leaving group ability: H2O, HO-, Br-, H-

A) H2O > Br- > HO- > H-
B) H2O > HO- > Br- > H-
C) HO- > Br- > H2O > H-
D) HO- > H- > H2O > Br-

A

A

Good leaving groups are weak bases, which are the conjugates of strong acids. Leaving groups must also be stable once they leave the molecule. H2O is, by far, the most stable leaving group and will be extremely unreactive once it leaves the molecule through heterolysis. Br- is the conjugate base of HBr; HO- is the conjugate base of water. HBr is a much stronger acid than water, so Br- is a better leaving group than HO-. Finally, hydride (H-) is a very poor leaving group because it is extremely unstable in solution.

How well did you know this?
1
Not at all
2
3
4
5
Perfectly
69
Q

Rank the following in order of decreasing oxidation state: amine, carboxylic acid, aldehyde, alkane

A) Aldehyde, amine, alkane, carboxylic acid
B) Carboxylic acid, aldehyde, amine, alkane
C) Carboxylic acid, amine, aldehyde, alkane
D) Alkane, amine, aldehyde, carboxylic acid

A

B

Carboxylic acids are the second most oxidized form of carbon (only carbon dioxide is more oxidized). In carboxylic acids, the carbon atom has three bonds to oxygen. In aldehydes, the carbon atom has two bonds to oxygen. In amines, the carbon atom has one bond to nitrogen. In an alkane, the carbon only has bonds to other carbons and hydrogens.

How well did you know this?
1
Not at all
2
3
4
5
Perfectly
70
Q

If cinnamaldehyde was treated with LiAlH4, what reaction would occur?

A) Reduction, resulting in a primary alcohol
B) Oxidation, resulting in a carboxylic acid
C) An acid-base reaction, resulting in a diol
D) No reaction would occur

A

A

All that we need to know about cinnamaldehyde is that is is an aldehyde, and therefore will be reduced by a strong reducing agent like LiAlH4 to a primary alcohol.

How well did you know this?
1
Not at all
2
3
4
5
Perfectly
71
Q

If 2-butanol was treated with dichromate, what reaction would occur?

A) Reduction, resulting in the formation of butene
B) Oxidation, resulting in the formation of butanoic acid
C) Oxidation, resulting in the formation of butanone
D) No reaction would occur

A

C

Because 2-butanol is a secondary alcohol, oxidation by a strong oxidizing agent like dichromate will result in a ketone, butanone.

How well did you know this?
1
Not at all
2
3
4
5
Perfectly
72
Q

If 1-hexanol was treated with pyridinium chlorochromate, what would the end product be?

A) 2-hexanol
B) 2-hexanone
C) Hexanal
D) Hexanoic acid

A

C

Pyridinium chlorochromate is a weak oxidizing agent, and will oxidize an alcohol to an aldehyde. Stronger oxidizing agents are required to convert a primary alcohol to a carboxylic acid.

73
Q

Sn1 reactions show first-order kinetics because:

A) The rate-limiting step is the first step to occur in the reaction
B) The rate-limiting step involves only one molecule
C) There is only one rate-limiting step
D) The reaction involves only one molecule

A

B

An Sn1 reaction is a first-order nucleophilic substitution reaction. It is called first-order because the rate-limiting step involves only one molecule. Choice A is true, but does not explain why Sn1 reactions have first-order kinetics; the rate-limiting step of an Sn2 reaction is also the first (and only) step of that reaction, but Sn2 reactions have second-order kinetics, not first-order. Choice C is a true statement as well, but again does not explain why the reaction is first-order. Finally, Choice D is incorrect because it is the rate-limiting step, not the reaction overall, that involves only one molecule.

74
Q

In a protic solvent, which of the following halogens would be the best nucleophile?

A) Br-
B) Cl-
C) F-
D) I-

A

D

In a protic solvent, the protons in solution can attach to the nucleophile, decreasing nucleophilicity. The larger the nucleophile, and the stronger its conjugate acid, the stronger the nucleophile will be. Of the options, I- will therefore be the strongest nucleophile because it is least likely to associate with the protons in solution.

75
Q

Treatment of (S)-2-bromobutane with sodium hydroxide results in the production of a compound with an (R) configuration. This reaction has most likely taken place through:

A) An Sn1 mechanism
B) An Sn2 mechanism
C) An Sn1 and Sn2 mechanism in sequence
D) An Sn1 and Sn2 mechanism simultaneously

A

B

In this reaction, there has been an inversion of stereochemistry. The most likely explanation for this is that the reaction proceeded by an Sn2 mechanism. Inversion of stereochemistry is a hallmark of Sn2 reactions, whereas racemization is a hallmark of Sn1.

76
Q

Which of the following solvents would be LEAST useful for a nucleophile-electrophile reaction?

A) H2O
B) CH3CH2OH
C) CH3SOCH3
D) CH3CH2CH2CH2CH2CH3

A

D

To carry out a nucleophile-electrophile reaction, the nucleophile must be able to dissolve in the solvent. Nucleophiles are nearly always polar, and often carry a charge. Polar solvents are therefore preferred for these reactions. Hexane is a nonpolar solvent and will not be useful for a nucleophile-electrophile reaction.

77
Q

Aldehydes are generally more reactive than equivalent ketones to nucleophiles. This is likely due to differences in:

A) Steric hinderance
B) Leaving group ability
C) Resonance stabilization
D) Electron-withdrawing character

A

A

Aldehydes have one alkyl group connected to the carbonyl carbon, whereas ketones have two. This creates more steric hindrance in ketones, which lowers their reactivity to nucleophiles. Ketones are also less reactive because their carbonyl carbon has less positive charge character; the additional alkyl group can donate electron density - the opposite of Choice D - which decreases the electrophilicity of the compound.

78
Q

Which conversion between carboxylic acid derivatives is NOT possible by a nucleophilic reaction?

A) Carboxylic acid to ester
B) Ester to carboxylic acid
C) Anhydride to amide
D) Ester to anhydride

A

D

Remember there is a hierarchy to the reactivity of carboxylic acid derivatives that dictates how reactive they are toward nucleophilic attack. In order from highest to lowest, this hierarchy is anhydrides > carboxylic acids and esters > amides. In practical terms, this means that derivatives of higher reactivity can form derivatives of lower reactivity but not vice versa. Nucleophilic attack of an ester cannot result in a corresponding anhydride because anhydrides are more reactive than esters.

79
Q

Alcohols has higher boiling points than their analogous hydrocarbons because:

A) The oxygen atoms in alcohols have shorter bond lengths
B) Hydrogen bonding is present in alcohols
C) Alcohols are more acidic than their analogous hydrocarbons
D) Alcohols can be oxidized to ketones

A

B

Alcohols have higher boiling points than their analogous hydrocarbons as a result of their polarized O-H bonds, in which oxygen is partially negative and hydrogen is partially positive. This enables the oxygen atoms of other alcohol molecules to be attracted to the hydrogen, forming a hydrogen bond. Heat is required to overcome these hydrogen bonds, thereby increasing the boiling point. The analogous hydrocarbons do not form hydrogen bonds and, therefore, vaporize at lower temperatures. Choice A is irrelevant; oxygen’s bond length is not a factor in determining a substance’s boiling point. Choices C and D are true statements, but are also irrelevant to boiling point determination.

80
Q

Tertiary alcohols are oxidized with difficulty because:

A) There is no hydrogen attached to the carbon with the hydroxyl group
B) There is no hydrogen attached to the alpha-carbon of the carbonyl
C) Tertiary alcohols contain hydroxyl groups with no polarization
D) They are relatively inert

A

A

Tertiary alcohols can be oxidized but only under extreme conditions because their substrate carbons do not have spare hydrogens to give up. Alcohol oxidation involves the removal of such a hydrogen so that carbon can instead make another bond to oxygen. If no hydrogen is present, a carbon-carbon bond must be cleaved, which requires a great deal of energy and will, therefore, occur only under extreme conditions. Choice B is incorrect because alcohols are not carbonyl-containing compounds and would more properly describe a carbonyl-containing compound that is unable to form an enolate. Choice C is incorrect because the hydroxyl group of the tertiary carbon is still polarized. Choice D is a false statement; tertiary alcohols are still involved in other reactions, such as Sn1 reactions.

81
Q

The IUPAC name of this molecule is:

              /\    /
            /    \/
      HO      |
                 OH

A) ethane-1,2-diol
B) propane-1,2-diol
C) dimethanol
D) dipropanol

A

B

Remember, diols are named after the parent alkane, with the position of the alcohols indicated, and ending in the suffix -diol. Here the carbon chain is three carbons, with a hydroxyl group on carbons 1 and 2. Thus, the name is propane-1,2-diol.

82
Q

Which of the following correctly lists methanol, isobutyl alcohol, and propanol by decreasing boiling point?

A) Methanol > isobutyl alcohol > propanol
B) Isobutyl alcohol > methanol > propanol
C) Isobutyl alcohol > propanol > methanol
D) Methanol > propanol > isobutyl alcohol

A

C

All else being equal, boiling points increase with increasing size of the alkyl chain because of increased van der Waals attractions. Isobutyl alcohol has the largest alkyl chain and will thus have the highest boiling point; methanol has the smallest chain and will thus have the lowest boiling point.

83
Q

Which of the following correctly lists hexanol, phenol, and cyclohexane by increasing acidity of the hydroxyl hydrogen?

A) Phenol < hexanol < cyclohexanol
B) Cyclohexanol < hexanol < phenol
C) Cyclohexanol < phenol < hexanol
D) Phenol < cyclohexanol < hexanol

A

B

Phenols have significantly more acidic hydroxyl hydrogens than other alcohols because of resonance stabilization of the conjugate base, so this will be the most acidic hydroxyl hydrogen. The acidity of hexanol and cyclohexanol are close, but the hydroxyl hydrogen of hexanol is slightly more acidic because the ring structure of cyclohexanol is slightly electron-donating, which makes its hydroxyl hydrogen slightly less acidic.

84
Q

Which of the following will convert CH3CH2CH2OH to CH3CH2CHO?

I. CrO3
II. PCC
III. K2Cr2O7

A) I only
B) II only
C) I and III only
D) I, II, and III

A

B

CH3CH2CH2OH is 1-propanol, a primary alcohol. The desired end product, CH3CH2CHO, is propanal, an aldehyde. Of the available options, the only reactant capable of oxidizing primary alcohols to aldehydes is pyridinium chlorochromate (PCC). Chromic trioxide and dichromate salts will both oxidize primary alcohols to carboxylic acids.

85
Q

Which of the following will convert cyclohexanol to cyclohexanone?

I. Chromium trioxide
II. Pyridinium chlorochromate
III. Sodium dichromate

A) I only
B) II only
C) I and III only
D) I, II, and III

A

D

Cyclohexanol is a secondary alcohol, so any of the oxidizing agents listed will convert it to a ketone.

86
Q

Successfully converting 3-phenylpropanol to 3-phenylpropanoic acid by the Jones oxidation requires the oxidizing agent, the solvent, and:

A) Dilute sulfuric acid
B) Dilute sodium hydroxide
C) Anhydrous conditions
D) High temperature

A

A

Acidic conditions, provided by dilute sulfuric acid, are required to complete the Jones oxidation. This reaction is carried out in aqueous conditions. While heat may speed up the reaction, high temperatures are not required for this reaction.

87
Q

Treating 2-methyl-1-propanol with methylsulfonyl chloride in base, followed by reaction with pyridinium chlorochromate, and a final step in strong acid, will give an end product of:

A) 2-methyl-1-propanol
B) 2-methylpropanal
C) 2-methylpropanoic acid
D) 2-methyl-1-propane

A

A

Methylsulfonyl chloride serves as a protecting group for alcohols, which are converted into mesylates. Reacting with this reagent before continuing with what would normally be an oxidation reaction keeps the alcohol from reacting; when the protecting group is then removed using strong acid, the resultant product is the same as the initial reactant. Neither of the oxidation products in Choices B or C, nor the reduction product in Choice D, will be formed.

88
Q

Reaction of 1-phenylethanone with ethylene glycol, also known as ethane-1,2-diol, in aqueous H2SO4 will result in the formation of:

A) A ketal
B) A carboxylic acid
C) An aldehyde
D) A hemiacetal

A

A

This reaction will create a ketal. This is the first step of the protection of aldehydes or ketones using dialcohols.

89
Q

In order to convert phenols into hydroxyquinones, how many steps of oxidation or reduction are required?

A) 1 oxidation step
B) 2 oxidation steps
C) 1 reduction step
D) 2 reduction steps

A

B

In order to convert phenols into hydroxyquinones, they must first be converted into quinones through an oxidation step; a second oxidation step is required to further oxidize quinones to hydroxyquinones.

90
Q

The conversion of ubiquinone and ubiquinol requires what type of reaction?

A) Condensation
B) Oxidation
C) Reduction
D) Hydrolysis

A

C

The reaction that converts ubiquinone into ubiquinol is a reduction reaction in which two ketones are reduced to two hydroxyl groups.

91
Q

Which of the following will convert a cyclic acetal to a carbonyl and a dialcohol?

A) Aqueous acid
B) LiAlH4
C) CrO3
D) Acetone

A

A

An acetal can be converted to a carbonyl and a dialcohol by treatment with aqueous acid. This is the final step when using alcohols as protecting groups, called deprotection.

92
Q

All of the following are true with respect to carbonyls EXCEPT:

A) The carbonyl carbon is electrophilic
B) The carbonyl carbon is electron-withdrawing
C) A resonance structure of the functional group places a positive charge on the carbonyl carbon
D) The pi electrons are mobile and are pulled toward the carbonyl carbon

A

D

The reactivity of the carbonyl can be attributed to the difference in electronegativity between the carbon and oxygen atoms. The more electronegative oxygen atom attracts the bonding electrons and is therefore electron-withdrawing. Thus, the carbonyl carbon is electrophilic. One resonance structure of the carbonyl pushes the pi electrons onto the oxygen, resulting in a positively charged carbonyl carbon.

93
Q

Order the following compounds by increasing boiling point: butane, butanol, butanone.

A) Butanol < butane < butanone
B) Butane < butanone < butanol
C) Butanone < butane < butanol
D) Butane < butanol < butanone

A

B

Assuming the length of the carbon chain remains the same, the alkane consistently has the lowest boiling point. The boiling point of the ketone is elevated by the dipole in the carbonyl. The boiling point of the alcohol is elevated further by hydrogen bonding.

94
Q

The formation of α-d-glucopyranose from β-d-glucopyranose is called:

A) Glycosidation
B) Mutarotation
C) Enantiomerization
D) Racemization

A

B

Mutarotation is the interconversion between anomers of a compound. Enantiomerization and racemization, choices (C) and (D), mean the same thing as each other: the formation of a mirror-image or optically inverted form of a compound. Glycosidation, choice (A), is the addition of a sugar to another compound.

95
Q

Hemiacetals and hemiketals usually keep reacting to form acetals and ketals. Why is it difficult to isolate hemiacetals and hemiketals?

I. These molecules are unstable
II. The hydroxyl group is rapidly protonated and lost as water under acidic conditions, leaving behind a reactive carbocation
III. The molecules are extremely basic and react rapidly with one another

A) I only
B) I and II only
C) II and III only
D) I, II, and III

A

B

Hemiacetals and hemiketals are usually short-lived because the -OH group will rapidly become protonated in acidic conditions and is lost as water, leaving behind a carbocation that is very susceptible to attack by an alcohol. Once the alcohol has been added, the acetal or ketal becomes more stable because the newly added group is less likely to become protonated and leave as compared to -OH.

96
Q

In a hemiacetal, the central carbon is bonded to:

A) -OH, -OR, -H, and -R
B) -H, -OR, -OR, and -R
C) -OH, -OR, -R, and -R
D) -OR, -OR, -R, and -R

A

A

A hemiacetal is a molecule in which one equivalent of alcohol has been added to a carbonyl (-OR) and the carbonyl oxygen has been protonated (-OH). Otherwise, there is the same alkyl group (-R) and hydrogen atom (-H) as the parent aldehyde. Choice B describes an acetal, C a hemiketal, and D a ketal.

97
Q

In a reaction between hydrogen cyanide, butyraldehyde, and ethylmethylketone, which compounds will come together to form the major product?

A) Butyraldehyde and hydrogen cyanide
B) Ethylmethylketone and butryaldehyde
C) Hydrogen cyanide and ethylmethylketone
D) No reaction will occur

A

A

Although both the aldehyde and ketone listed will be reactive with the strongly nucleophilic hydrogen cyanide, aldehydes are slightly more reactive toward nucleophiles than ketones for steric reasons, so the aldehyde and HCN will form major product (which will be a cyanohydrin).

98
Q

Which of the following describe(s) pyridinium chlorochromate (PCC)?

I. An oxidant that can form aldehydes from primary alcohols
II. An oxidant that can completely oxidize primary alcohols
III. An oxidant that can completely oxidize secondary alcohols

A) I only
B) I and II only
C) I and III only
D) I, II, and III

A

C

PCC is a mild anhydrous oxidant that can oxidize primary alcohols to aldehydes, and secondary alcohols to ketones. It is not strong enough to oxidize alcohols or aldehydes to carboxylic acids.

99
Q

To form a geminal diol, which of the following could attack a carbonyl carbon?

A) Hydrogen peroxide
B) Water
C) Potassium dichromate
D) Ethanol

A

B

In a hydration reaction, water adds to a carbonyl, forming a geminal diol - a compound with two hydroxyl groups on the same carbon. Hydrogen peroxide and potassium dichromate are oxidizing agents that can convert an aldehyde to a carboxylic acid. Ethanol will react with a carbonyl compound to form an acetal or a ketal, if excess ethanol is available.

100
Q

In a reaction between ammonia and glutaraldehyde, what is the major product?

A) An imine
B) A cyanohydrin
C) A semicarbazone
D) A hydrazone

A

A

Ammonia, or NH3, will react with an aldehyde like glutaraldehyde to form an imine. This is a condensation and a substitution reaction, as the C = O of the carbonyl will be replaced with a C ≡ N bond.

101
Q

Which of the following can be used to reduce a ketone to a secondary alcohol?

A) CrO3
B) KMnO4
C) LiAlH4
D) Ag2O

A

C

Hydrides like LiAlH4 and NaBH4 are reducing agents; as such, they will reduce aldehydes and ketones to alcohols. The other reagents listed are oxidizing agents, which will not act on a ketone.

102
Q

Imines naturally tautomerize to form:

A) Oximes
B) Hydrazones
C) Semicarbazones
D) Enamines

A

D

During tautomerization, the double bond between the carbon and nitrogen in an imine is moved to lie between two carbons. This results in an enamine - a compound with a double bond and an amine.

103
Q

The reaction below is an example of:

    ^              --->            ^
  /   \\       <---------       /   \\
       O                    HO

A) Esterification
B) Tautomerization
C) Elimination
D) Dehydration

A

B

Keto <—> enol

Tautomerization is the interconversion of two isomers in which a hydrogen and a double bond are moved. The keto and enol tautomers of aldehydes and ketones are common examples of tautomers seen on Test Day. Note that the equilibrium lies to the left because the keto form is more stable. Esterification is the formation of esters from carboxylic acids and alcohols. Elimination is a reaction in which a part of a reactant is removed and a new multiple bond is introduced. Dehydration is a reaction in which a molecule of water is eliminated.

104
Q

Why does the equilibrium between keto and enol tautomers lie far to the keto side?

I. The keto form is more thermodynamically stable
II. The enol form is lower energy
III. The enol form is more thermodynamically stable

A) I only
B) III only
C) I and II only
D) II and III only

A

A

The keto-enol equilibrium lies far to the keto side because the keto form is significantly more thermodynamically stable than the enol form. This thermodynamic stability stems from the fact that the oxygen is more electronegative than the carbon, and the keto tautomer puts more electron density around the oxygen than the enol tautomer. If the enol tautomer is less thermodynamically stable, it is also higher energy than the keto tautomer.

105
Q

The aldol condensation is an example of which reaction type(s)?

I. Dehydration
II. Cleavage
III. Nucleophilic addition

A) I only
B) I and III only
C) II and III only
D) I, II, and III

A

B

The aldol condensation is both a dehydration reaction because a molecule of water is lost, and a nucleophilic addition reaction because the nucleophilic enolate attacks and bonds to the carbonyl carbon.

106
Q

When reacted with ammonia (NH3) at 200℃, which enolate of a carbonyl-containing compound would predominate?

A) Kinetic enolate
B) Thermodynamic enolate
C) Neither enolate; they would present in roughly equal proportions
D) Neither enolate; these reaction conditions would not form either enolate

A

B

At high temperatures and with a weak base like NH3, the thermodynamic enolate will be favored. The reaction proceeds slowly with the weak base, giving the kinetic enolate time to interconvert to the more stable thermodynamic enolate.

107
Q

Which of the following compounds would be most reactive toward a nucleophile?

A) Pentanal
B) 3-pentanone
C) Pentane
D) 2-nonanone

A

A

Aldehydes are generally more reactive than ketones because the additional alkyl group of a ketone is sterically hindering; this alkyl group is also electron-donating, destabilizing the carbanion intermediate. The carbonyl carbon is highly electrophilic; alkanes lack any significant electrophilicity.

108
Q

ɑ-hydrogens of a ketone are acidic due to:

I. Resonance stabilization
II. The electron-withdrawing properties of the alkyl groups
III. The electronegative carbonyl oxygen

A) I only
B) I and III only
C) II and III only
D) I, II, and III

A

B

When ɑ-carbons are deprotonated, the negative charge is resonance stabilized in part by the electronegative carbonyl oxygen, which is electron-withdrawing. Alkyl groups are actually electron-donating, which destabilizes carbanion intermediates; this invalidates statement II.

109
Q

Which of the following is considered a tautomer of the imine function group?

A) Cyanohydrin
B) Hydrazone
C) Enamine
D) Semicarbazone

A

C

All of the answer choices are nitrogen-containing functional groups, but only enamines are tautomers of imines. Imines contain a double bond between a carbon and a nitrogen; enamines contain a double bond between two carbons as well as an amine.

110
Q

When succinaldehyde is treated with lithium diisopropylamide (LDA), it:

I. Becomes more nucleophilic
II. Becomes less nucleophilic
III. Generates a carbanion

A) I only
B) II only
C) I and III only
D) II and III only

A

C

When succinaldehyde (or any aldehyde or ketone with ɑ-hydrogens) is treated with a strong base like lithium diisopropylamide (LDA), it forms the more nucleophilic enolate carbanion.

111
Q

Which of the following best describes the final product of an aldol condensation?

A) 1,3-dicarbonyl
B) 1,2-dicarbonyl
C) ɑ-β-unsaturated carbonyl
D) β,ɤ-unsaturated carbonyl

A

C

Aldol condensations contain two main steps. In the first step, the ɑ-carbon of an aldehyde or ketone is deprotonated, generating the enolate carbanion. This carbanion can then attack another aldehyde or ketone, generating the aldol. In the second step, the aldol is dehydrated, forming a double bond. This double bond is between the ɑ- and β-carbons, so the molecule is an ɑ,β-unsaturated carbonyl.

112
Q

When benzaldehyde is reacted with acetone, which will act as the nucleophile?

A) Benzaldehyde, after addition of strong acid
B) Benzaldehyde, after reaction with strong base
C) Acetone, after addition of strong acid
D) Acetone, after reaction with strong base

A

D

Because benzaldehyde lacks a ɑ-proton, it cannot be reacted with base to form the nucleophilic enolate carbanion. Therefore, acetone will act as as our nucleophile. In order to perform this reaction, which is an aldol condensation, acetone will be reacted with a strong base - not a strong acid - in order to extract the ɑ-hydrogen and form the enolate anion, which will act as a nucleophile.

113
Q

3-hydroxybutanal can be formed by the reaction of:

A) Methanol in diethyl ether
B) Ethanal in base, then in acid
C) Butanal in strong acid
D) Methanal and ethanal in catalytic base

A

B

This is an example of an aldol condensation, but stopped after aldol formation (before dehydration). After the aldol is formed using strong base, the reaction may be halted by the addition of acid. Butanal in strong acid would be likely to deprotonate without gaining the hydroxyl group. Methanal in diethyl ether would not be reactive because diethyl ether is not a strong enough base to abstract the ɑ-hydrogen. Reaction of the two aldehydes methanal and ethanal in catalytic base would form 3-hydroxypropanal (which would dehydrate to form propanal), not 3-hydroxybutanal.

114
Q

The catalytic production of dihydroxyacetone and glyceraldehyde 3-phosphate (2-hydroxy-3-oxopropyl dihydrogen phosphate) from fructose-1,6-bisphosphate ({[(2S,3S,4S,5R)-3,4-dihydroxy-5-[(phosphonooxy)methyl]oxolan-2-yl]methoxy}phosphonic acid) is what type of reaction?

A) Aldol condensation
B) Retro-aldol reaction
C) Dehydration
D) Nucleophilic attack

A

B

The nomenclature in this question is well above what one needs to be able to draw on the MCAT; however, we can discern that we are forming a ketone and an aldehyde from a single molecule. The hallmark of a reverse aldol reaction is the breakage of a carbon-carbon bond, forming two aldehydes, two ketones, or one of each. In an aldol condensation, we would expect to form a single product by combining two aldehydes, two ketones, or one of each. A dehydration reaction should release a water molecule, rather than breaking apart a large organic molecule into two small molecules. A nucleophilic attack should feature the formation of a bond between a nucleophile and an electrophile; again, we would not expect to break apart a large organic molecule into two smaller molecules. Note that simply noting how many reactants and products are present in the reaction is sufficient to determine the answer.

115
Q

Carboxylic acids have higher boiling points than their corresponding alcohols primarily because:

A) Molecular weight is increased by the additional carboxyl group
B) The pH of the compound is lower
C) Acid salts are soluble in water
D) Hydrogen bonding is much stronger than in alcohols

A

D

The boiling points of compounds depend on the strength of the attractive forces between molecules. In both alcohols and carboxylic acids, the major form of intermolecular attraction is hydrogen bonding; however, hydrogen bonding is much stronger in carboxylic acids as compared to alcohols because carboxylic acids are more polar and the carbonyl also contributes to hydrogen bonding in addition to the hydroxyl group. The stronger hydrogen bonds elevate the boiling points of carboxylic acids compared to alcohols. Boiling points also depend on molecular weight, but in this case, the difference in molecular weight is insignificant compared to the effect of hydrogen bonding. Choices B and C are both true but do not explain the difference in boiling points.

116
Q

Which of the following carboxylic acids will be the most acidic?

A) CH3CHClCH2COOH
B) CH3CH2CCl2COOH
C) CH3CH2CHClCOOH
D) CH3CH2CH2COOH

A

B

The acidity of carboxylic acids is significantly increased by the presence of highly electronegative functional groups. Their electron-withdrawing effect increases the stability of the carboxylate anion, favoring proton dissociation. This effect increases as the number of electronegative groups on the chain increases, and it also increases as the distance between the acid functionality and electronegative group decreases. This answer has two halogens bonded to it at a smaller distance from the carbonyl group compared to the other answers.

117
Q

Which of the following molecules could be classified as a soap?

A) CH3(CH2)17CH2COOH
B) CH3COOH
C) CH3(CH2)19COO–Na+
D) CH3COO–Na+

A

C

Soap is a salt of a carboxylate anion with a long hydrocarbon tail. Choices A and B are not salts of anionic compounds. Choice D is sodium acetate, which is a salt but does not contain the long hydrocarbon tail needed to be considered a soap.

118
Q

Carboxylic acids can be reacted in one step to form all of the following compounds EXCEPT:

A) Esters
B) Amides
C) Alkenes
D) Alcohols

A

C

Carboxylic acids cannot be converted into alkenes in one step. Esters are formed in nucleophilic acyl substitution reactions with alcohols. Amides are formed by nucleophilic acyl substitution reactions with ammonia. Alcohols may be formed using a variety of reducing agents. To form alkenes, carboxylic acids may be reduced to alcohols, which can then be transformed into alkenes by elimination in a second step.

119
Q

The reduction of a carboxylic acid using lithium aluminum hydride will yield what final product?

A) An aldehyde
B) An ester
C) A ketone
D) An alcohol

A

D

Lithium aluminum hydride (LiAlH4 or LAH) is a strong reducing agent. LAH can completely reduce carboxylic acids to primary alcohols. Aldehydes are intermediate products of this reaction. The other compounds are not created through the reduction of a carboxylic acid.

120
Q

Which of the following is true with respect to a micelle in a hydrophilic environment?

A) The interior is hydrophilic
B) The structure, as a whole, is hydrophobic
C) It is composed of short-chain fatty acids with polar heads
D) It can dissolve nonpolar molecules deep in its core

A

D

Micelles are self-assembled aggregates of soap in which the interior is composed of long hydrocarbon (fatty) tails, which can dissolve nonpolar molecules. The outer surface is covered with carboxylate groups, which makes the overall structure water-soluble. Soaps, in general, are salts of long-chain hydrocarbons with carboxylate head groups.

121
Q

In the presence of an acid catalyst, the major product of butanoic acid and 1-pentanol is:

A) 1-butoxy-1-pentanol
B) Butyl pentanoate
C) 1-pentoxy-1-butanol
D) Pentyl butanoate

A

D

The reaction described is esterification, in which the nucleophile oxygen atom of 1-pentanol attacks the electrophilic carbonyl carbon of butanoic acid, ultimately displacing water to form pentyl butanoate. The acid catalyst is regenerated from 1-pentanol’s released proton. Choice A reverses the carbon chains, considering the butyl tail to be the esterifying group. Ethers do not form under these conditions.

122
Q

The α-hydrogen of a carboxylic acid is:

I. More acidic than the hydroxyl hydrogen
II. Less acidic than the hydroxyl hydrogen
III. Very acidic, as organic compounds go

A) I only
B) II only
C) I and III only
D) II and III only

A

D

The ɑ-hydrogen of a carboxylic acid is relatively acidic as far as organic compounds go, due to resonance stabilization. However, the hydroxyl hydrogen is significantly more acidic because it is able to share the negative charge resulting from deprotonation between the electronegative oxygen atoms in the functional group.

123
Q

The reaction of formic acid with sodium borohydride will yield what final product?

A) An aldehyde
B) A carboxylic acid
C) A ketone
D) An alcohol

A

B

The reaction of formic acid, which is a simple carboxylic acid, with sodium borohydride, which is a mild reducing agent, will result in no reaction, and therefore will result in maintenance of the carboxylic acid. Sodium borohydride is too mild to reduce carboxylic acids, and therefore cannot produce the primary alcohols that lithium aluminum hydride, a strong reducing agent, would.

124
Q

The intramolecular reaction of 5-aminopentanoic acid through nucleophilic acyl substitution would result in a(n):

A) Anhydride
B) Lactone
C) Lactam
D) Carboxylic acid

A

C

5-aminopentanoic acid contains a carboxylic acid and an amine. If this molecule undergoes intramolecular nucleophilic acyl substitution, it will form a cyclic amide. These molecules are called lactams. Lactones are cyclic esters, not amides.

125
Q

Butanoic anhydride can be produced by the reaction of butanoic acid with which of the following compounds?

A) Butanoic acid
B) Ethanoic acid
C) Butanol
D) Methanal

A

A

Butanoic anhydride is an anhydride with two butane R groups. Anhydrides are produced by the reaction of two carboxylic acids with the loss of a water molecule. Therefore, butanoic anhydride would be produced by the reaction of two molecules of butanoic acid.

126
Q

Nucleophilic acyl substitution is favored by:

I. Basic solution
II. Acidic solution
III. Leaving groups that are strong bases

A) I only
B) II only
C) I and II only
D) I, II, and III

A

C

Nucleophilic acyl substitutions are favored in basic solution, which makes the nucleophile more nucleophilic; in acidic solution, which makes the electrophile more electrophilic; and by good leaving groups. However, strong bases do not make good leaving groups; weak bases do.

127
Q

The reaction of ammonia with caprylic acid (octanoic acid), found in coconuts, would produce a(n):

A) Ester
B) Anhydride
C) Alcohol
D) Water molecule

A

D

Based on its name, caprylic acids must be a carboxylic acid. The reaction between a carboxylic acid and ammonia (NH3) would produce an amide - which is not one of the options listed. Instead, we should take a look at the type of reaction occurring. The production of an amide from a carboxylic acid and ammonia occurs through a condensation reaction in which a molecule of water is removed as a leaving group.

128
Q

Which of the following would be the best method of producing methyl propanoate?

A) Reacting propanoic acid and methanol in the presence of a mineral acid
B. Reacting methanoic acid and propanol in the presence of a mineral acid
C. Reacting propanoic anhydride with an aqueous base
D. Reacting propanoic acid with an aqueous base

A

A

Methyl propanoate is an ester; it can be synthesized by reacting a carboxylic acid with an alcohol in the presence of acid. Here, the parent chain is propanoate, and the esterifying group is a methyl group. Choice B reverses the nomenclature, and would therefore form propyl methanoate. The other reactions listed would not form esters.

129
Q

Each of the acyl compounds listed below contains a six-membered ring EXCEPT:

A) δ-lactam
B) Cyclohexane carboxylic acid
C) γ-butyrolactone
D) The anhydride formed from intramolecular ring closure of pentanedioic acid

A

C

This question requires knowledge of the nomenclature of cyclic molecules. A δ-lactam has a bond between the nitrogen and the fourth carbon away from the carbonyl carbon. This ring will have six elements: the nitrogen, the carbonyl carbon, and the four carbons in between. Cyclohexane carboxylic acid has cyclohexane, a six-membered cycloalkane. The anhydride formed from pentanedioic acid will have the five carbons in the parent chain and one oxygen atom closing the ring, meaning there are still six elements. γ-butyrolactone will have five elements because it contains a bond between the ester oxygen and the third carbon away from the carbonyl carbon. The five elements will be the oxygen, the carbonyl carbon, and the three carbons in between.

130
Q

Which of the following would be most reactive toward nucleophiles?

A) Propyl ethanoate
B) Propanoic acid
C) Propanamide
D) Propanoic anhydride

A

D

With the same R groups, steric influence is the same in each case, so we can therefore rely solely on electronic effects. When this is all that is taken into account, reactivity toward nucleophiles is highest for anhydrides, followed by esters and carboxylic acids, then amides.

131
Q

Which of the following would react most readily with a carboxylic acid to form an amide?

A) Methylamine
B) Triethylamine
C) Diphenylamine
D) Ethylmethylamine

A

A

Methylamine would react readily to form an amide. The less substituted the nucleophile, the easier it will be for the nucleophile to attack the carbonyl carbon and form the amide. In fact, triethylamine would not be able to form an amide at all because it does not have a hydrogen to lose while attaching to the carbonyl carbon.

132
Q

If propanamide were treated with water, what product(s) would be observed?

A) Propanamide
B) Propanoic acid
C) Equal concentrations of propanamide and propanoic acid
D) Propyl propanoate

A

A

Propanamide is an amide; as such, it is the least reactive of the carboxylic acid derivatives discussed in the chapter. Without strong acid or base, propanamide will not be able to undergo nucleophilic acyl substitution and no reaction will occur.

133
Q

β-lactams are:

I. Cyclic forms of the least reactive type of carboxylic acid derivative
II. More reactive than their straight-chain counterparts
III. Molecules with high levels of ring strain

A) I only
B) II only
C) II and III only
D) I, II, and III

A

D

β-lactams are amides in the form of four-membered rings; amides are generally the least reactive type of carboxylic acid derivative. β-lactams experience significant ring strain from both eclipsing interactions (torsional strain) and angle strain, and are therefore more susceptible to hydrolysis than the linear form of the same molecule.

134
Q

The acid-catalyzed conversion of propyl ethanoate to benzyl ethanoate is likely:

A) Reduction
B) Hydrolysis
C) Transesterification
D) Oxidation

A

C

As far as we can tell, we are converting one ester to another in this reaction. The fact that this reaction is acid-catalyzed should confirm the suspicion that this is transesterification.

135
Q

Which reactant could be combined with butanol to form butyl acetate?

A) (CH3CO)2O and catalytic acid
B) (CH3CH2CO)2O and catalytic acid
C) CH3CH2CONH2 and catalytic acid
D) CH3CONH2 and catalytic acid

A

A

In order to prepare butyl acetate from butanol, we need to perform a nucleophilic acyl substitution reaction. If the product is an ester, we need to start with a reactant that is more reactive than the ester itself, or the reaction will not proceed. Anhydrides are more reactive than esters, but amides are less reactive. Reaction with propanoic anhydride would result in butyl propanoate.

136
Q

Why should esterification reactions NOT be carried out in water?

A) Carboxylic acids, from which esters are made, are generally insoluble in water
B) The polar nature of water overshadows the polar nature of the leaving group
C) The extensive hydrogen bonding of water interferes with the nucleophilic addition mechanism
D) Water molecules would hydrolyze the desired products back into the parent carboxylic acid

A

D

The presence of water in an esterification reaction would likely revert some of the desired esters back into carboxylic acids. Small carboxylic acids, like formic or acetic acid, are easily dissolved in water. The polarity of water plays little role in affecting the leaving group; if anything, water can be used to increase electrophilicity of the carbonyl carbon by protonating the carbonyl oxygen. Finally, this is a nucleophilic substitution mechanism, not a nucleophilic addition mechanism. Further, hydrogen bonding would likely augment the reaction.

137
Q

Which of the following amino acids does not have an L-enantiomer?

A) Cysteine
B) Threonine
C) Glutamic acid
D) Glycine

A

D

Glycine’s R group is a hydrogen atom; this amino acid is therefore achiral because the central carbon is not bonded to four different substituents. The other amino acids are all chiral and therefore have both L- and D-enantiomers.

138
Q

Which of the following would be formed if methyl bromide were reacted with phthalimide and followed by hydrolysis with an aqueous base?

A) C2H5NH2
B) CH3NH2
C) (C2H5)3N
D) (CH3)4N+Br–

A

B

This reaction is similar to the Gabriel synthesis. Phthalimide acts as nucleophile, the methyl carbon acts as an electrophile, and bromide acts as the leaving group. Therefore, the reaction between methyl bromide and phthalimide results in the formation of methyl phthalimide. Subsequent hydrolysis then yields methylamine.

139
Q

Which of the following amino acids contain(s) sulfur?

I. Cysteine
II. Serine
III. Methionine

A) I only
B) I and III only
C) II and III only
D) I, II, and III

A

B

Cysteine is well known for containing a sulfur atom because it is able to form disulfide bridges; however, methionine also contains a sulfur atom in its R group.

140
Q

Nylon, a polyamide, is produced from hexanediamine and a substance X. This substance X is most probably a(n):

A) Amine
B) Carboxylic acid
C) Ketone
D) Alcohol

A

B

An amide is formed from an amine and a carboxyl group or its acyl derivatives. In this question, an amine is already given; the compound to be identified must be an acyl compound. The only acyl compound among the choices given is a carboxylic acid.

141
Q

Intermediates in the Strecker synthesis include all of the following nitrogen-containing functional groups EXCEPT a(n):

A) Nitrile
B) Imine
C) Amide
D) Amine

A

C

During the Strecker synthesis, ammonia attacks a carbonyl, forming an imine. This imine is attacked by cyanide, forming a amine and a nitrile. Amide bonds are formed between amino acids, but do not appear during the Strecker synthesis.

142
Q

Why is the C–N bond of an amide planar?

I. It has partial double-bond character
II. It is sp3-hybridized
III. It has some sp2 character

A) I only
B) II only
C) I and II only
D) I and III only

A

D

One resonance structure of a C - N bond in an amide has the double bond between the C and N, not between the C and O. Thus, the C - N bond of an amide has some sp2 character, and sp2-hybridized atoms exhibit planar geometry.

143
Q

Which of the primary methods of amino acid synthesis results in an optically active solution?

A) The Strecker synthesis only
B) The Gabriel synthesis only
C) Both the Strecker and Gabriel syntheses
D) Neither the Strecker nor the Gabriel syntheses

A

D

Both the Strecker and Gabriel syntheses contain planar intermediates, which can be attacked from either side by a nucleophile. This results in a racemic mixture of enantiomers, and the solution will therefore be optically inactive.

144
Q

During the Gabriel synthesis, phthalimide serves as the:

A) Nucleophile
B) Base
C) Leaving group
D) Electrophile

A

A

During the Gabriel synthesis, phthalimide attacks a secondary carbon in diethyl bromomalonate. The secondary carbon is the electrophile, and bromide is the leaving group.

145
Q

Each of the following reaction types occurs during the Gabriel synthesis EXCEPT:

A) Decarboxylation
B) Nucleophilic substitution
C) Dehydration
D) Hydrolysis

A

C

The Gabriel synthesis includes two nucleophilic substitution steps, followed by hydrolysis and decarboxylation. Dehydration - the loss of a water molecule - is not part of this reaction.

146
Q

At physiological pH, which two forms of phosphoric acid have the highest concentrations?

A) H3PO4 and H2PO4-
B) H2PO4- and HPO4^2-
C) HPO4^2- and PO4^3-
D) PO4^3- and H3PO4

A

B

The pka2 of phosphoric acid is close to physiological pH; therefore, [H2PO4-] ≈ [HPO4^2-] at this pH.

147
Q

In aqueous solution, pyrophosphate will likely:

A) Form insoluble complexes
B) Be stable and inert
C) Degrade into inorganic phosphate
D) Decrease the polarity of the solvent

A

C

Pyrophosphate is unstable in aqueous solution and will degrade to form two equivalents of inorganic phosphate. The solvent is water, which should retain its polarity regardless of the presence of solutes. Pyrophosphate and inorganic phosphate are small, charged molecules which are relatively soluble.

148
Q

What would be the charge of aspartic acid at pH 7?

A) Neutral
B) Negative
C) Positive
D) There is not enough information to answer the question

A

B

The amino acid in question is aspartic acid, which is an acidic amino acid because it contains an extra carbonyl group. At neutral pH, both of the carbonyl groups are ionized, so there are two negative charges on the molecule. Only one of the charges is neutralized by the positive charge on the amino group, so the molecule has an overall negative charge.

149
Q

When a bond is created between two nucleotide triphosphates in DNA synthesis, the small molecule released from this reaction is:

A) Pyrophosphate
B.) Inorganic phosphate
C) ATP
D) Organic phosphate

A

A

As DNA is synthesized, it forms phosphodiester bonds, releasing pyrophosphate, PPi. Pyrophosphate is an inorganic phosphate-containing molecule, but it is not the single phosphate group commonly referred to as inorganic phosphate. The DNA molecule itself is referred to as an organic phosphate.

150
Q

The hydrogens of phosphoric acid have pKa values that:

A) Allow high buffering capacity over a small pH range
B) Allow moderate buffering capacity over a large pH range
C) Allow low buffering capacity over a small pH range
D) Do not allow buffering

A

B

Phosphoric acid has three hydrogens with pKa values spread across the pH range. This allows some degree of buffering over almost the entire standard pH range from 0 to 14.

151
Q

IR spectroscopy is most useful for distinguishing:

A) Double and triple bonds
B) C–H bonds
C) Chirality of molecules
D) Composition of racemic mixtures

A

A

Infrared spectroscopy is most useful for distinguishing between different functional groups. Almost all organic compounds have C - H bonds so except for fingerprinting a compound, these absorptions are not useful. Little information about the optical properties of a compound can be obtained by IR spectroscopy.

152
Q

Oxygen (O2) does not exhibit an IR spectrum because:

A) It has no molecular motions
B) It is not possible to record IR spectra of a gaseous molecule
C) Molecular vibrations do not result in a change in the dipole moment of the O2 molecule
D) Molecular oxygen contains four lone pairs overall

A

C

Because molecular oxygen is homonuclear (composed of only one element) and diatomic, there is no net charge in its dipole moment during vibration or rotation; in other words, the compound does not absorb in a measurable way in the infrared region. IR spectroscopy is based on the principle that, when the molecule vibrates or rotates, there is a change in dipole moment. Choice A is incorrect because oxygen does have molecular motions; they are just not detectable in IR spectroscopy. Choice B is incorrect because it is possible to record the IR of a gaseous molecule as long as it shows a change in its dipole moment when it vibrates. Choice D is incorrect because lone pairs do not have an effect on the ability to generate an IR spectrum of a compound.

153
Q

If IR spectroscopy were employed to monitor the oxidation of benzyl alcohol to benzaldehyde, which of the following would provide the best evidence that the reaction was proceeding as planned?

A) Comparing the fingerprint region of the spectra of starting material and product
B) Noting the change in intensity of the peaks corresponding to the benzene ring
C) Noting the appearance of a broad absorption peak in the region of 3100–3500 cm^–1
D) Noting the appearance of a strong absorption in the region of 1750 cm^–1

A

D

In this reaction, the functional group is changing from a hydroxyl to an aldehyde. This means that a sharp peak will appear around 1750 cm^-1, which corresponds to the carbonyl functionality. Choice C is the opposite of what occurs; the reaction will be characterized by the disappearance of the O-H peak at 3100 to 3500 cm^1, not its appearance. Comparing the fingerprint regions will provide evidence that a reaction is occurring, but it is not as useful for knowing that the reaction that occurred was indeed the one that was desired.

154
Q

Which of the following chemical shifts could correspond to an aldehydic proton signal in a 1H–NMR spectrum?

A) 9.5 ppm
B) 7.0 ppm
C) 11.0 ppm
D) 1.0 ppm

A

A

The peak at 9.5 ppm corresponds to an aldehydic proton. This signal lies downfield because the carbonyl oxygen is electron-withdrawing and deshields the proton. Choice C corresponds to a carboxyl proton and is even further downfield because the acidic proton is deshielded to a greater degree than the aldehydic proton. Choice B corresponds to aromatic protons. Choice D is characteristic of an alkyl proton on an sp3-hybridized carbon.

155
Q

The isotope 12C is not useful for NMR because:

A) It is not abundant in nature
B) Its resonances are not sensitive to the presence of neighboring atoms
C) It has no magnetic moment
D) The signal-to-noise ratio in the spectrum is too low

A

C

This isotope has no magnetic moment and will therefore not exhibit resonance with an applied magnetic field. Nuclei with odd mass numbers (1H, 11B, 13C, 15N, 19F, and so on) or those with an even mass number but an odd atomic number (2H, 10B) will have a nonzero magnetic moment.

156
Q

In 1H–NMR, splitting of spectral lines is due to:

A) Coupling between a carbon atom and protons attached to that carbon atom
B) Coupling between a carbon atom and protons attached to adjacent carbon atoms
C) Coupling between adjacent carbon atoms
D) Coupling between protons on adjacent carbon atoms

A

D

Spin-spin coupling (splitting) is due to influence on the magnetic environment of one proton by protons on the adjacent atom. These protons are three bonds away from each other. Splitting in other NMR spectra can include coupling with carbon atoms, but not in 1H-NMR.

157
Q

Compared to IR and NMR spectroscopy, UV spectroscopy is preferred for detecting:

A) Aldehydes and ketones
B) Unconjugated alkenes
C) Conjugated alkenes
D) Aliphatic acids and amines.

A

C

Most conjugated alkenes have an intense ultraviolet absorption. Aldehydes, ketones, acids, and amines all absorb in the ultraviolet range. However, other forms of spectroscopy (mainly IR and NMR) are more use for precise identification. Isolated alkenes can rarely be identified by UV spectroscopy.

158
Q

Considering only the 0 to 4.5 ppm region of a 1H–NMR spectrum, how could ethanol and isopropanol be distinguished?

A. They cannot be distinguished from 1H–NMR alone
B. A triplet and quartet are observed for ethanol, whereas a doublet and septet are observed for isopropanol
C. A triplet and quartet are observed for isopropanol, whereas a doublet and septet are observed for ethanol
D. The alcohol hydrogen in ethanol will appear within that region, whereas the alcohol hydrogen in isopropanol will appear downfield of that region

A

B

The region in question often gives information about the types of alkyl groups present. Specifically, ethanol will give a characteristic triplet for the methyl group (which is coupled to -CH2-) and a quartet for -CH2- (which is coupled to the methyl group). Isopropanol will have a septet for the -CH- group (which is coupled to both methyl groups combined) and a doublet for the two methyl groups (which are coupled to -CH-). In both cases, the proton in the alcohol does not participate in coupling. The alcohol hydrogen likely lies downfield for both compounds because it is bonded to such an electronegative element.

159
Q

Before absorbing an ultraviolet photon, electrons can be found in:

A) The HOMO only
B) The LUMO only
C) Both the HOMO and the LUMO
D) Neither the HOMO nor the LUMO

A

A

The HOMO is the highest occupied molecular orbital. Only after absorbing ultraviolet light is an electron excited from the HOMO to the LUMO, the lowest unoccupied molecular orbital.

160
Q

In an IR spectrum, how does extended conjugation of double bonds affect the absorbance band of carbonyl (C=O) stretches compared with normal absorption?

A) The absorbance band will occur at a lower wavenumber
B) The absorbance band will occur at a higher wavenumber
C) The absorbance band will occur at the same wavenumber
D) The absorbance band will disappear

A

A

Carbonyl groups (C=O) in conjugation with double bonds tend to absorb at lower wavenumbers because the delocalization of pi electrons causes the C=O bond to lose double-bond character, shifting the stretching frequency closer to C-O stretches. Remember that higher-order bonds tend to have higher absorption frequencies, so loss of double-bond character would decrease the absorption frequency of the group.

161
Q

Wavenumber is directly proportional to:

A) Wavelength
B) Frequency
C) Percent transmittance
D) Absorbance

A

B

Wavenumber (1/λ) is directly proportional to frequency (c/λ). It is inversely proportional to wavelength and has no proportionality to percent transmittance or absorbance.

162
Q

Two enantiomers will:

A) Have identical IR spectra because they have the same functional groups
B) Have identical IR spectra because they have the same specific rotation
C) Have different IR spectra because they are structurally different
D) Have different IR spectra because they have different specific rotations

A

A

Enantiomers will have identical IR spectra because they have the same functional groups and will therefore have the exact same absorption frequencies. Enantiomers have opposite specific rotations, but specific rotation actually has no effect on the IR spectrum.

163
Q

In a molecule containing a carboxylic acid group, what would be expected in a 1H–NMR spectrum?

A. A deshielded hydrogen peak for the hydroxyl hydrogen, shifted left
B. A deshielded hydrogen peak for the hydroxyl hydrogen, shifted right
C. A shielded hydrogen peak for the hydroxyl hydrogen, shifted left
D. A shielded hydrogen peak for the hydroxyl hydrogen, shifted right

A

A

The oxygen of the hydroxyl group will deshield the hydroxyl hydrogen, shifting it downfield, or left. Hydrogens in carboxylic acids can have some of the most downfield absorbances, around 10.5 to 12 ppm.

164
Q

The coupling constant, J, is:

A) The value of n + 1 when determining splitting in NMR spectra
B) Measured in parts per million (ppm)
C) Corrected for by calibration with tetramethylsilane
D) A measure of the degree of splitting caused by other atoms in the molecule

A

D

The coupling constant is a measure of the degree of splitting introduced by other atoms in a molecule, and is the frequency of the distance between subpeaks. It is measured in hertz. The coupling constant is independent of the value of n + 1, and is not changed by calibration with tetramethylsilane.

165
Q

The IR spectrum of a fully protonated amino acid would likely contain which of the following peaks?

I. A sharp peak at 1750 cm–1
II. A sharp peak at 3300 cm–1
III. A broad peak at 3300 cm–1

A) I only
B) I and II only
C) II and III only
D) I, II, and III

A

B

Amino acids in their fully protonated form contain all three of the peaks that should be memorized for Test Day: C-O, N-H, and O-H. While statements I and II correctly give the peaks for the C=O bond (sharp peak at 1750 cm^-1) and the N-H bond (sharp peak at 3300 cm^-1), the peak for the O-H bond is in the wrong place. In a carboxylic acid, the C=O bond withdraws electron density from the O-H bond, shifting the absorption frequency down to about 3000 cm^1. Statement III is therefore incorrect.

166
Q

Fractional distillation under atmospheric pressure would most likely be used to separate which of the following compounds?

A) Methylene chloride (boiling point of 40°C) and water (boiling point of 10°C)
B) Ethyl acetate (boiling point of 77°C) and ethanol (boiling point of 80°C)
C) Aniline (boiling point of 184°C) and benzyl alcohol (boiling point of 205°C)
D) Aniline (boiling point of 184°C) and water (boiling point of 100°C)

A

B

Fractional distillation is the most effective procedure for separating two liquids that boil within a few degrees of each other. Ethyl acetate and ethanol boil well within 25℃ of each other and thus would be good candidates for fractional distillation. Fractional distillation could also be used for the liquids in Choice C, but would require lower pressures because of their high boiling points.

167
Q

Which of the following compounds would be most effective in extracting benzoic acid from a diethyl ether solution?

A) Tetrahydrofuran
B) Aqueous hydrochloric acid
C) Aqueous sodium hydroxide
D) Water

A

C

By extracting with sodium hydroxide, benzoic acid will be converted to its sodium salt, sodium benzoate. Sodium benzoate, unlike its conjugate acid, will dissolve in an aqueous solution. The aqueous layer simply has to be acidified afterward to retrieve benzoic acid. Choice A is incorrect because diethyl ether and tetrahydrofuran are both nonpolar and are miscible. Hydrochloric acid will not form benzoic acid into a soluble salt. Finally Choice D is incorrect because protonated benzoic acid has limited solubility in water.

168
Q

Which of the following would be the best procedure for extracting acetaldehyde from an aqueous solution?

A A single extraction with 100 mL of ether
B) Two successive extractions with 50 mL portions of ether
C) Three successive extractions with 33.3 mL portions of ether
D) Four successive extractions with 25 mL portions of ether

A

D

It is more effective to perform four successive extractions with small amounts of ether than to perform one extraction with a large amount of ether.

169
Q

What would be the effect on the Rf values if the thin-layer chromatography (TLC) described below were run with hexane rather than ether as the eluent?

Compound Retardation Factor in Ether
Benzyl alcohol 0.10
Benzyl acetate 0.26
p-Nitrophenol 0.23
1-Naphthalenemethanol 0.40

A) No effect
B) Increase tenfold
C) Double
D) Decrease

A

D

Hexane is less polar than ether and, therefore, is less likely to displace polar compounds absorbed to the silica gel. This would decrease the distance these polar compounds would travel, decreasing Rf values.

170
Q

If the compounds below were separated by column chromatography with ether on silica gel, which would elute first?

Compound Retardation Factor in Ether
Benzyl alcohol 0.10
Benzyl acetate 0.26
p-Nitrophenol 0.23
1-Naphthalenemethanol 0.40

A) Benzyl alcohol
B) Benzyl acetate
C) p-Nitrophenol
D) 1-Naphthalenemethanol

A

D

In column chromatography, as in TLC, the less polar compound travels most rapidly. This means that 1-naphthalenemethanol, with the highest Rf value, would travel most rapidly and would be the first to elute from the column.

171
Q

Suppose an extraction with methylene chloride (density = 1.33g/mL) is performed, with the desired compound initially in brine (density = 1.04g/mL). In a separatory funnel, which layer will be the organic layer?

A. Top layer
B. Bottom layer
C. No layers are observed; methylene chloride and brine are completely miscible.
D. More information is needed to answer the question.

A

B

Because methylene chloride is denser than brine (salt water), the organic layer will settle at the bottom of the funnel. Methylene chloride is nonpolar, so it cannot mix with brine.

172
Q

Silica gel is often used in thin-layer chromatography. What property does silica gel probably possess that makes it useful for this purpose?

A) Acidity
B) Polarity
C) Specifically sized pores
D) Aqueous solubility

A

B

Silica gels are polar. Polarity is used to selectively attract specific solutes within a nonpolar solvent phase. Although silica gels have other properties, this is the most important to TLC.

173
Q

A mixture of sand, benzoic acid, and naphthalene in ether is best separated by:

A) Filtration, followed by acidic extraction, followed by recrystallization
B) Filtration, followed by basic extraction, followed by evaporation
C) Extraction, followed by distillation, followed by gas chromatography
D) Filtration, followed by size-exclusion column chromatography, followed by extraction

A

B

In this question, three substances must be separated using a combination of techniques. The first step should be the most obvious: remove the sand by filtration. The remaining compounds - benzoic acid and naphthalene - are still dissolved in ether. If the solution is extracted with aqueous base, the benzoate anion is formed and becomes dissolved in the aqueous layer, while naphthalene, a nonpolar compound, remains in the ether. Finally, evaporation of the ether will yield purified naphthalene.

174
Q

Simple distillation could be used to separate which of the following compounds?

A) Toluene (boiling point of 111°C) and water (boiling point of 100°C)
B) Naphthalene (boiling point of 218°C) and butyric acid (boiling point of 163°C)
C) Propionaldehyde (boiling point of 50°C) and acetic acid (boiling point of 119°C)
D) Benzene (boiling point of 80°C) and isopropyl alcohol (boiling point of 83°C)

A

C

This is the only option that would be effectively separated by simply distillation. Choice B would require vacuum distillation because the boiling points are over 150℃. In Choice A and D, the boiling points are within 25℃ of each other and would therefore require fractional distillation in order to be separated.

175
Q

In order to separate a biological effector from solution, which chromatographic technique would be the most effective?

A) Thin-layer chromatography
B) Ion-exchange chromatography
C) Affinity chromatography
D) Size-exclusion chromatography

A

C

Affinity chromatography, using the target for the biological effector or a specific antibody, would work best in this case. It will specifically bind the protein of interest and keep it in the column.

176
Q

Given a solution of insulin (molecular weight = 5.8 kD) and titin (molecular weight = 3816 kD), which chromatographic technique would be the most effective for separating out usable molecules of titin?

A) Thin-layer chromatography
B) Ion-exchange chromatography
C) Affinity chromatography
D) Size-exclusion chromatography

A

D

Because the solution is composed of a much larger molecule and a much smaller molecule, size-exclusion chromatography would effectively remove the smaller insulin molecule into the fraction retained in the column and allow the titin to be eluted. Affinity chromatography could also be used, but comes with a risk of rendering the titin unusable; the eluent run through an affinity chromatography column often binds to the target molecule.

177
Q

The gas eluent in gas chromatography and the liquid eluent in paper chromatography are examples of which component of these systems?

A) Stationary phase
B) Mobile phase
C) Column
D) Fraction

A

B

Each of these is the mobile phase of the system, in which the solutes are dissolved and move. The stationary phase in gas chromatography is usually a crushed metal or polymer; the stationary phase in paper chromatography is paper.

178
Q

During gravity filtration, a student forgets to heat the solution before running it through the filter. After capturing the filtrate, the student analyzes the sample via infrared (IR) spectroscopy and finds none of the desired product in the filtrate. What likely occurred to the student’s product?

A) The product degraded because of a prolonged filtration time
B) The product evaporated with collection of the filtrate
C) The product precipitated and is present in the residue
D) The product was dissolved in the solvent

A

C

Warm or hot solvent is generally used in gravity filtration to keep the desired product soluble. This allows the product to remain in the filtrate, which can then be collected. In this case, the student likely used solvent that was too cold, and the product crystallized out. The product should be present in the residue.

179
Q

Lactoferrin, a milk protein, is a valuable antimicrobial agent that is extracted from pasteurized, defatted milk utilizing a column containing a charged resin. This is an example of which of the following chromatographic techniques?

A) Thin-layer chromatography
B) Ion-exchange chromatography
C) Affinity chromatography
D) Size-exclusion chromatography

A

B

Because the lactoferrin proteins are likely to be charged, as is the resin described in the question, this is an example of ion-exchange chromatography. The charged protein molecules will stick to the column, while the remainder of the milk washes through and can later be washed off of the column and collected.